Mitä aika on?

Anonyymi-ap

Kvanttimekaniikan puolella on esiintynyt papereita, jonka mukaan aika syntyy lomittumisesta ja se on olemassa vain eristetyn systeemin sisällä. Ulkopuolisen näkökulmasta systeemi on staattinen ja ajaton.

Ymmärtääkseni edellinen väittämä liittyy enemmän säieteoriaan, mutta osaakohan joku avata asiaa vähän enemmän? Säeteorian kanssa kilpaileva Loop Quantum Gravity (LQG) teorian mukaan aika on myös emergentti-ilmiö, mutta eri mielessä.

Asiaa käsittelevä artikkeli:
https://www.scientificamerican.com/article/what-is-spacetime-really-made-of/

Erityisesti kaipaisin kommenttia artikkeliin, joka väittää, että ajan syntyminen lomittumisen seurauksena olisi testattu:
https://medium.com/the-physics-arxiv-blog/quantum-experiment-shows-how-time-emerges-from-entanglement-d5d3dc850933

Aiheeseen liittyy nähdäkseni myös kysymys monimaailmoista. Vaikuttaisi, että systeemin sisällä on olemassa lukuisia rinnakkaisia mahdollisia historioita ja systeemin ulkopuolinen voi mallintaa sitä oikein vain laskemalla kaikkien mahdollisten historioiden yhteisvaikutuksen. Tämä kuulostaa science fictionilta, mutta kun kvanttimekaniikassa mallinnetaan elektroniseen dipolimomenttiin liittyvää g-kerrointa teemme juuri tätä. Feynmanin diagrammit esittävät lukuisia mahdollisia elektronin ja virtuaalihiukkasten reaktioita ja saamme johdettua vakion oikein käsittämättömällä tarkkuudella, kun oletamme, että kaikki mahdolliset historiat tapahtuivat. Tätä on kutsuttu kvanttimekaniikan ällistyttävämmäksi ennustukseksi. Oletteko samaa mieltä tästä tulkinnasta? Asia on kuvattu:
https://www.youtube.com/watch?v=7UwigY4SjKY

Voi siis olla, ettei aika ole ainoastaan suhteellista vaan elämme rinnakkaisten historioiden verkostossa, missä kaikki mahdollisuudet toteutuvat ja mahdolliset historiat voivat jopa vuorovaikuttaa keskenään, jos ne johtavat yhteen ja samaan lopputilaan. Silti systeemin ulkopuolelta katsottuna aika on emergentti ominaisuus. Tätä monimaailma-näkemystä on kuvattu videossa:
https://www.youtube.com/watch?v=GlOwJWJWPUs

Fyysikot liikkuvat tässä nähdäkseni jonkun suuren oivalluksen parissa, joka voi olla keskeistä koko todellisuuden ymmärtämisessä. Erilaiset hologrammiset universumimallit lyövät oman kortensa kekoon, mutta loppujen lopuksi kukaan ei kai ole onnistunut yhdistämään kaikkia vihjeitä tyydyttävällä tavalla. Vinkkejä ajan luonteesta näyttäisi olevan paljon aina kvanttimekaniikan malleista mustien aukkojen olemukseen.

37

338

    Vastaukset

    Anonyymi (Kirjaudu / Rekisteröidy)
    5000
    • Anonyymi

      "Aiheeseen liittyy nähdäkseni myös kysymys monimaailmoista."
      Jos tuon kvanttifysiikan monimaailmatulkinnan korvaa sillä idealla että potentiaalisuus on yhtä todellista kuin empiirisesti koettu aineellinen todellisuus niin kokonaisuus on huomattavasti selkeämpi.

      Potentiaalisuus on ainakin makrotasolla aina suurempi kuin se mikä voi fysikaalisena todellisuutena aktualisoitua.

      Esim. fysiikassa ja myös ihan arjen tason kokemuksessa tunnetaan potentiaalinergian käsite joka tarkoittaa sitä että johonkin fysikaaliseen kohteeseen on varastoitunut energiaa (esim. viritetty jousi) jonka aktualisoitumisen tapa ja suunta on epävarmaa siihen asti kunnes se varastoitunut energia pääsee purkautumaan.

      Schrödingerin aaltiyhtälö ei liity fysikaalisiin aaltoihin vaan aaltomaisesti käyttäytyviin todennäköisyysaaaltoihin joiden mukaan yritetään määritellä hiukkasten liikkeiden tulevaisuus tilastollisella tasolla jolloin kyseessä on nimenomaan epävarmuus joka seurauksena hiukkasilla on enemmän apausasteita toisin kuin esim. kiinteillä biljadipalloilla jotka käyttäytyvät suhteellisen deterministisesti vaikka kuluvat käytössä ajan kuluessa.

      Varmuus ja determinismi pätee oikeastaan vain kiinteiden jäykkien kappaleiden tasolla ainakin jossain määrin mutta koska luonnontieteissä on perinteisesti pyritti mahdollisimman suureen varmuuteen ja tarkkuuteen niin tietyssä mielessä monimaailmatulkinta (ossa kaikki mahdollinen toteutuu omassa deterministisessä maailmassaan) voi tuntua joidenkin mielestä houkuttelevalta vaihtoehdolta jos tutkijoiden epävarmuuden sietokyky on heikko ja ainoa työkalu on matemaattinen hahmotus determinististen kaavojen avulla.

      Monimaailmamalli käytännössä tuottaisi valtavan määrän versumeita jokaisen hiukkastaon vuorovaikutuksen seurauksena ja jo pelkästään logiikan Reductio ad absurdum periaatteen mukaan pitäisi hylätä täysin järjettömänä ja epärealistisena tulkintana.

      Einsteinin blokkiuniversumin viitekehyksesä ajalla ei ole merkitystä ja sama koskee kaikkea sellaista ontologiaa jossa ei ole ajan suuntaa, entropiaa eikä selkeää identiteettiä joka alkaa ilmetä vasta kemian ja molekyylien ja makrokappaleiden tasoilla joissa voidaan tunnistaa selkeitä rakenteita.

      ...

      Teoreettinen fysiikkatiede monien muidenkin tieteenalojen ohella on ollut umpikujassa ja n. 100 vuotta eikä loppua ole näkyvissä ainakaan nykyisen akateemisen valtavirran puitteissa. ;D

      B

      • Anonyymi

        ” Esim. fysiikassa ja myös ihan arjen tason kokemuksessa tunnetaan potentiaalinergian käsite joka tarkoittaa sitä että johonkin fysikaaliseen kohteeseen on varastoitunut energiaa (esim. viritetty jousi) jonka aktualisoitumisen tapa ja suunta on epävarmaa siihen asti kunnes se varastoitunut energia pääsee purkautumaan.”

        Osaatko avata ajatustasi enemmän? En osaa nähdä noissa esimerkeissä mitään epävarmuutta suuntien suhteen.


      • Anonyymi

        Monimaailma-tulkinta on hyvin luontaisen yksinkertainen. En näe perustetta pitää sitä järjettömänä. Nähdäkseni se on selityksistä yksinkertaisin.

        Kööpenhaminan tulkinta ja pilottiaalto-selitykset sen sijaan tuntuvat aika väkinäisiltä. Etenkin, kun meillä on esimerkkejä tilanteista, missä eri historiat vaikuttaisivat reagoivan keskenään kuten aloituksessa mainitussa g-kertoimen teoreettisessa johtamisessa.


      • Anonyymi
        Anonyymi kirjoitti:

        Monimaailma-tulkinta on hyvin luontaisen yksinkertainen. En näe perustetta pitää sitä järjettömänä. Nähdäkseni se on selityksistä yksinkertaisin.

        Kööpenhaminan tulkinta ja pilottiaalto-selitykset sen sijaan tuntuvat aika väkinäisiltä. Etenkin, kun meillä on esimerkkejä tilanteista, missä eri historiat vaikuttaisivat reagoivan keskenään kuten aloituksessa mainitussa g-kertoimen teoreettisessa johtamisessa.

        "Osaatko avata ajatustasi enemmän? En osaa nähdä noissa esimerkeissä mitään epävarmuutta suuntien suhteen."

        Tuo viritetty jousi oli vain yksi esimerkki potentiaalisesta varastoituneesta energiasta. Sen suhteen ei ole sinänsä paljoa epämääräisyttä ja potentiaalisuutta paitsi sen aktualisoitumisen ajankohdan suhteen koska kyseessä on kiinteään makroaineeseen liittyvä esimerkki.

        Se epävarmuus kuitenkin alkaa vaikuttaa kun siirrytään sinne kenttien ja hiukkasten ja toisaalta myös elollisen ja elävän luonnon tasolle jotka myös usein hahmotetaan jotenkin satunnaisina vaikka kyse ei ole aidosta satunnaisuudesta vaan siitä että niiden kausaaliketjuja ei voida määritellä fysikaaliselta tasolta lähtien koska ne kykenevät aloittamaan omia kausaaliketjujaan fysikaalisen aineen tasolla mistä seurauksena ihmisen kulttuurin tasolla ovat mm. tekniikan kehitys ja soveltaminen joka on tietysti edelleen kiistanalainen kysymys koska tämä kulttuuri on perinteisesti voimakkaasti sitoutunut deterministiseen fysikalistiseen maailmankuvaan joka on pohjimmiltaan pelkkä ehdollistettu ajatustottumus samalla tavalla kuin uskonnotkin.

        ...

        "Monimaailma-tulkinta on hyvin luontaisen yksinkertainen. En näe perustetta pitää sitä järjettömänä. Nähdäkseni se on selityksistä yksinkertaisin."

        Käytännössä monimaailmatulkinnan suosio johtunee siitä että keskitytään vain siihen omaan universumiin ja unohdetaaan ne vaihtoehtoiset universumit jolloin ei tarvitse opetella mitään uutta ja 1800-luvun lopun deterministinen malli pysyy käytännössä muuttumattomana.

        Mikrotasolla tuo monimaailman postulointi voi tuntua uskottavalta mutta jos sen yrittää hahmottaa esim. arjen makrotasolla niin siitä seuraa että on lukematon määrä sinuja ja minuja omassa maailmassaan tekemässä kaikkea mahdollista mitä ylipäätänsä voi kuvitella eikä millään valinnoilla ole mitään merkitystä koska jos valitset jotain tässä maailmassa niin ne kaikki muutkin mahdolliset valinnat totetutuvat eli jossain maailmassa olet esim. Suomen presidentti tai Universumin keisari yms. :D

        Monimaailmatulkinta on tyyppiesimerkki siitä kuinka tiukasti jotkut haluavat takertua joihinkin metafysiikan tason oletuksiin jotka ovat aikoinaan omaksuneet maailmankuvansa perusteiksi eikä se koske pelkästään materialisteja vaan periaatteessa kaikkia jotka ovat samaistuneet johonkin uskomusjärjetelmään josta on tullut olennainen osa heidän omaa identiteettiään eli jos se uskomusjärjestelmä kyseeenalaistetaan niin se koetaan samalla hyökkäyksenä sitä omaa identiteettiä vastaan. Kyse on hyvin yleisestä inhimillisestä ominaisuudesta joka käytännössä voi jumittaa yksilön sekä myös tieteen ja kulttuurin kehitystä.

        "Kööpenhaminan tulkinta ja pilottiaalto-selitykset sen sijaan tuntuvat aika väkinäisiltä. Etenkin, kun meillä on esimerkkejä tilanteista, missä eri historiat vaikuttaisivat reagoivan keskenään kuten aloituksessa mainitussa g-kertoimen teoreettisessa johtamisessa."

        Teor. fysiikassa matemaattinen mielikkuvitus on päästetty valloilleen eli kyse on pohjimmiltaan metafysiikan ja filosofian tason villistä spekulaatiosta joka on piilotettun matemaattisen formalismin alle. Mitää tahansa teoriaa voi yllpitää loputtomiin esim. postuloimalla uusia entieteettejä kuten uusia hiukkasia tai säännönmukaisuuksia jos teoria alkaa näyttää empiirisen tason suhteen epäuskottavalta.

        https://en.wikipedia.org/wiki/Fudge_factor

        Minusta kyseessä on väärä todellisuuden perushahmotus jota ei helposti huomata erikoistuneiden omissa lokeroissaan operovien tieteenalojen tasolla mutta jonka voi huomata laajemman näkökulman ja viitekehyksen puitteissa eli kokonaistodellisuuden tasolla jossa tosin joudutaan tinkimään sekä tarkkuudesta että matemaattisen soveltamisen yleispätevyydestä. Se mikä voi näyttää ns. "lillukanvarsien" tasolla oikealta ja järkevältä voi kokonaisuuden kannalta olla ääliömäisyyden huipentumaa. :D

        Miljoonan vuoden päästä ehkä ihmiskunta voi ymmärtää vähän nykyistä enemmän...

        B


      • Anonyymi
        Anonyymi kirjoitti:

        "Osaatko avata ajatustasi enemmän? En osaa nähdä noissa esimerkeissä mitään epävarmuutta suuntien suhteen."

        Tuo viritetty jousi oli vain yksi esimerkki potentiaalisesta varastoituneesta energiasta. Sen suhteen ei ole sinänsä paljoa epämääräisyttä ja potentiaalisuutta paitsi sen aktualisoitumisen ajankohdan suhteen koska kyseessä on kiinteään makroaineeseen liittyvä esimerkki.

        Se epävarmuus kuitenkin alkaa vaikuttaa kun siirrytään sinne kenttien ja hiukkasten ja toisaalta myös elollisen ja elävän luonnon tasolle jotka myös usein hahmotetaan jotenkin satunnaisina vaikka kyse ei ole aidosta satunnaisuudesta vaan siitä että niiden kausaaliketjuja ei voida määritellä fysikaaliselta tasolta lähtien koska ne kykenevät aloittamaan omia kausaaliketjujaan fysikaalisen aineen tasolla mistä seurauksena ihmisen kulttuurin tasolla ovat mm. tekniikan kehitys ja soveltaminen joka on tietysti edelleen kiistanalainen kysymys koska tämä kulttuuri on perinteisesti voimakkaasti sitoutunut deterministiseen fysikalistiseen maailmankuvaan joka on pohjimmiltaan pelkkä ehdollistettu ajatustottumus samalla tavalla kuin uskonnotkin.

        ...

        "Monimaailma-tulkinta on hyvin luontaisen yksinkertainen. En näe perustetta pitää sitä järjettömänä. Nähdäkseni se on selityksistä yksinkertaisin."

        Käytännössä monimaailmatulkinnan suosio johtunee siitä että keskitytään vain siihen omaan universumiin ja unohdetaaan ne vaihtoehtoiset universumit jolloin ei tarvitse opetella mitään uutta ja 1800-luvun lopun deterministinen malli pysyy käytännössä muuttumattomana.

        Mikrotasolla tuo monimaailman postulointi voi tuntua uskottavalta mutta jos sen yrittää hahmottaa esim. arjen makrotasolla niin siitä seuraa että on lukematon määrä sinuja ja minuja omassa maailmassaan tekemässä kaikkea mahdollista mitä ylipäätänsä voi kuvitella eikä millään valinnoilla ole mitään merkitystä koska jos valitset jotain tässä maailmassa niin ne kaikki muutkin mahdolliset valinnat totetutuvat eli jossain maailmassa olet esim. Suomen presidentti tai Universumin keisari yms. :D

        Monimaailmatulkinta on tyyppiesimerkki siitä kuinka tiukasti jotkut haluavat takertua joihinkin metafysiikan tason oletuksiin jotka ovat aikoinaan omaksuneet maailmankuvansa perusteiksi eikä se koske pelkästään materialisteja vaan periaatteessa kaikkia jotka ovat samaistuneet johonkin uskomusjärjetelmään josta on tullut olennainen osa heidän omaa identiteettiään eli jos se uskomusjärjestelmä kyseeenalaistetaan niin se koetaan samalla hyökkäyksenä sitä omaa identiteettiä vastaan. Kyse on hyvin yleisestä inhimillisestä ominaisuudesta joka käytännössä voi jumittaa yksilön sekä myös tieteen ja kulttuurin kehitystä.

        "Kööpenhaminan tulkinta ja pilottiaalto-selitykset sen sijaan tuntuvat aika väkinäisiltä. Etenkin, kun meillä on esimerkkejä tilanteista, missä eri historiat vaikuttaisivat reagoivan keskenään kuten aloituksessa mainitussa g-kertoimen teoreettisessa johtamisessa."

        Teor. fysiikassa matemaattinen mielikkuvitus on päästetty valloilleen eli kyse on pohjimmiltaan metafysiikan ja filosofian tason villistä spekulaatiosta joka on piilotettun matemaattisen formalismin alle. Mitää tahansa teoriaa voi yllpitää loputtomiin esim. postuloimalla uusia entieteettejä kuten uusia hiukkasia tai säännönmukaisuuksia jos teoria alkaa näyttää empiirisen tason suhteen epäuskottavalta.

        https://en.wikipedia.org/wiki/Fudge_factor

        Minusta kyseessä on väärä todellisuuden perushahmotus jota ei helposti huomata erikoistuneiden omissa lokeroissaan operovien tieteenalojen tasolla mutta jonka voi huomata laajemman näkökulman ja viitekehyksen puitteissa eli kokonaistodellisuuden tasolla jossa tosin joudutaan tinkimään sekä tarkkuudesta että matemaattisen soveltamisen yleispätevyydestä. Se mikä voi näyttää ns. "lillukanvarsien" tasolla oikealta ja järkevältä voi kokonaisuuden kannalta olla ääliömäisyyden huipentumaa. :D

        Miljoonan vuoden päästä ehkä ihmiskunta voi ymmärtää vähän nykyistä enemmän...

        B

        Monimaailma tulkinta ei ole mitenkään suosittu asiaan perehtymättömien keskuudessa… Ja juurikin niiden lähinnä henkilökohtaisten mukavuustekijöiden vuoksi, mitä sinäkin toit esiin. Halutaan pitää kiinni ajatuksesta, että on vain yksi historia, vapaa tahto, aitoja valintoja sekä yksi versio sinusta. Nämä eivät kuitenkaan ole rationaalisia tai järjellisiä argumentteja itse ajatusta vastaan.

        Pointtiasi jonkun presidentti-haaveista en oikein ymmärtänyt. Kyllähän kvanttimekaniikan satunnaisuudellakin on rajansa - etenkin makromaailmassa. Kvanttimaailman satunnaisuudesta ei seuraa, että mikä vaan on mahdollista. Ja vaikka sinä olisit päässyt presidentiksi jossain rinnakkaistodellisuudessa, niin ei tällä ole mitään merkitystä tämän todellisuuden suhteen.

        Matematiikka on työkalu maailman selittämiseen. Se on tarkka - eikä siinä ole samoja epämääräisyys ongelmia kuin verbaalisissa selityksissä. Sillä mitä me pidämme uskottavana ei ole merkitystä. Pohjimmiltaan mallin hyvyys mitataan sillä miten tarkasti se mallintaa maailmaa, tekee ennustuksia ja se voidaan varmentaa mittauksin.

        Se, että teoriaan joudutaan tekemään ad-hoc lisäyksiä osoittaa sen heikkoutta. Parhaat ja menestyksekkäimmät teoriat ovat pohjaidealtaan yksinkertaisia. Kritiikkisi on kuitenkin aiheellinen, jos ajattelemme esimerkiksi pimeää ainetta. Se tuntuu ad-hoc lisäykseltä, jolla yritetään selittää, miksi malli ei ennusta havaintoja.

        Mutta menemme nyt tässä ohi aiheesta. Kenties kannattaisi pistää jokin toinen ketju. En tosin ihan saa kiinni siitä, mihin kritiikkisi eniten osuu.


      • Anonyymi
        Anonyymi kirjoitti:

        Monimaailma tulkinta ei ole mitenkään suosittu asiaan perehtymättömien keskuudessa… Ja juurikin niiden lähinnä henkilökohtaisten mukavuustekijöiden vuoksi, mitä sinäkin toit esiin. Halutaan pitää kiinni ajatuksesta, että on vain yksi historia, vapaa tahto, aitoja valintoja sekä yksi versio sinusta. Nämä eivät kuitenkaan ole rationaalisia tai järjellisiä argumentteja itse ajatusta vastaan.

        Pointtiasi jonkun presidentti-haaveista en oikein ymmärtänyt. Kyllähän kvanttimekaniikan satunnaisuudellakin on rajansa - etenkin makromaailmassa. Kvanttimaailman satunnaisuudesta ei seuraa, että mikä vaan on mahdollista. Ja vaikka sinä olisit päässyt presidentiksi jossain rinnakkaistodellisuudessa, niin ei tällä ole mitään merkitystä tämän todellisuuden suhteen.

        Matematiikka on työkalu maailman selittämiseen. Se on tarkka - eikä siinä ole samoja epämääräisyys ongelmia kuin verbaalisissa selityksissä. Sillä mitä me pidämme uskottavana ei ole merkitystä. Pohjimmiltaan mallin hyvyys mitataan sillä miten tarkasti se mallintaa maailmaa, tekee ennustuksia ja se voidaan varmentaa mittauksin.

        Se, että teoriaan joudutaan tekemään ad-hoc lisäyksiä osoittaa sen heikkoutta. Parhaat ja menestyksekkäimmät teoriat ovat pohjaidealtaan yksinkertaisia. Kritiikkisi on kuitenkin aiheellinen, jos ajattelemme esimerkiksi pimeää ainetta. Se tuntuu ad-hoc lisäykseltä, jolla yritetään selittää, miksi malli ei ennusta havaintoja.

        Mutta menemme nyt tässä ohi aiheesta. Kenties kannattaisi pistää jokin toinen ketju. En tosin ihan saa kiinni siitä, mihin kritiikkisi eniten osuu.

        "Monimaailma tulkinta ei ole mitenkään suosittu asiaan perehtymättömien keskuudessa… "

        Riippuu aika paljon siitä mitä sillä monimaailmalla oikeasti tarkoitetaan. Jos se monimaailma koskee lähinnä vain todellisuuden mikrotasoa ja lakkaa käytännön tasolla vaikuttamasta esim. dekoherenssin ja karkeistumisen seurauksena atomeja suuremmassa mittakaavassa niin se voi vaikuttaa ihan ok tulkinnalta muiden kvanttifysiikan tason tulkintojen ohella jotka mielestäni ovat kaikki jollain tavalla virheellisiä liian kapean näkökulman takia.

        Käsittääkseni on myös sellaisia monimaailman versioita joissa ne "maailmat" voivat vuorovaikuttaa keskenään ja sellaisia joissa se mahdollisuus ei ole sallittu.

        Itse olen sillä kannalla että se alkup. ja perimmäinen todellisuus liittyy jollain tavalla epälokaaliin prototietoisuuteen josta vähitellen alkoi erilaistumaan ja samalla ehdollistumaan sellainen todellisuus joka ilmenee atomeina, molekyinä ja makrokappaleina determinismin samalla kasvaessa kun erilaistuminen edistyy.

        Jos todellisuuden hahmottaa ensisijassa aaltojen ja kenttien kautta niin se perimmäinen todellisuus on epälokaali kokonaisuus jossa ne aallot ja kentät muodostavat joko konstruktiivisia tai destruktiivisia interferenssejä toistensa kanssa josta mallinnuksesta syntyy myös eräänlainen monimaailma mahdollisten havaitsijoiden/vuorovaikuttajien kannalta koska destruktiivinen interferenssi on vuorovaikutuksen puuttumista suhteessa havaitsijaan niin sellaiset kokonaisuudet ovat havaitsijan kannalta olemattomia ja vastaavastai vain konstuktiivisen havainto/vuorovaikutusinterferenssin tuottamat ilmiöt kuuluvat samaan maailmaan kulloinkin ko. havaitsijan kanssa ja kannalta.

        Koska erilaisilla ja myös samanlajisilla havaitsijoilla voi myös yksilötasolla olla erilaisia ominaisuuksia ja kykyjä niin käytännössä näin voi syntyä toisiinsa nähden rinakkaisia todellisuuksia havaitsijoille eli monimaailmojen kokonaisuus mikä ilmenee jos suhteessa siihen miten yksilöt kykenevät hahmottaamaan kokonaisuuksia joihin ovat perehtyneet verrattuna maallikkoihin eli huomataan eri asioita samasta kokonaisuudesta. Tässä mielessä myös esim. eri tieteenalojen ja ammattien asiantuntijat tavallaan elävät erilaisissa maailmoissa jotka eivät ole yhteismitallisia kaikilta osin.
        ...
        Kritiikkini kohdistuu lähinnä sellaiseen mekaaniseen ja kaavamaiseen monimaailmatulkintaan jossa maailmoja monistuu jatkuvasti vain sen takia että halutaan takertua universaalin determinismin ideaan.

        Mielestäni determinismi alkaa vaikuttaa vasta atomien tasolla ja on voimakkaammillaan molekyylien tasolla koska niiden paikka ja liike voidaan määritellä yksiselitteisesti ja ne ovat myös yksilöitävissä olevia entiteeettejä toisin kuin alkeishiukkasten ja kenttien tasot vaikka identiteetin määritelmä lakkaa toimimasta jos ajan eli muutokset ottaa huomioon eli perinteinen logiikka (A=A,A=B,B=C.,A=C jne. ) romahtaa jos ajan ottaa huomioon ja se toimii vain fiktiivisten kielellisten käsitteiden tasolla mistä on seurauksia myös matematiikan soveltamisen suhteen koska se logiikka että matematiikka ovat ihmisten kehittämiä loogisia konstruktioita jotka eivät sellaisenaan kerro mitään todellisuudesta vaan korkeintaan siitä miten yritämme hahmottaa sitä. ( "Jos vasara on ainoa työkalu niin kaikki mahdollinen alkaa näyttää naulamaiselta.")

        Se determinismi on myös sekä välttämätön että hyödyllinen aineen ominaisuus eliöiden ja varsinkin tietoisten ihmisten kaltaisten eliöiden tasolla koska se mahdollistaa deterministisesti käyttäytyvän aineen soveltamisen työkaluina ja tekniikkana mutta se ei tarkoita sitä että eliöt ja varsinkaan tietoiset oliot olisivat samalla tavalla niin tiukasti deterministisesti käyttäytyviä kuin se molekyylitason inertti aine. Tavallaan eliöiden kehot ovat yksilöllistymiseen pyrkivien tietoisten olioiden käyttövälineitä.

        Teorioiden ennusteiden ongelmaa olen käsitellyt muissa yhteyksissä ja samoin matematiikan ja logiikan suhdetta todellisuuteen joten ei niistä enempää.

        B


      • Anonyymi
        Anonyymi kirjoitti:

        "Monimaailma tulkinta ei ole mitenkään suosittu asiaan perehtymättömien keskuudessa… "

        Riippuu aika paljon siitä mitä sillä monimaailmalla oikeasti tarkoitetaan. Jos se monimaailma koskee lähinnä vain todellisuuden mikrotasoa ja lakkaa käytännön tasolla vaikuttamasta esim. dekoherenssin ja karkeistumisen seurauksena atomeja suuremmassa mittakaavassa niin se voi vaikuttaa ihan ok tulkinnalta muiden kvanttifysiikan tason tulkintojen ohella jotka mielestäni ovat kaikki jollain tavalla virheellisiä liian kapean näkökulman takia.

        Käsittääkseni on myös sellaisia monimaailman versioita joissa ne "maailmat" voivat vuorovaikuttaa keskenään ja sellaisia joissa se mahdollisuus ei ole sallittu.

        Itse olen sillä kannalla että se alkup. ja perimmäinen todellisuus liittyy jollain tavalla epälokaaliin prototietoisuuteen josta vähitellen alkoi erilaistumaan ja samalla ehdollistumaan sellainen todellisuus joka ilmenee atomeina, molekyinä ja makrokappaleina determinismin samalla kasvaessa kun erilaistuminen edistyy.

        Jos todellisuuden hahmottaa ensisijassa aaltojen ja kenttien kautta niin se perimmäinen todellisuus on epälokaali kokonaisuus jossa ne aallot ja kentät muodostavat joko konstruktiivisia tai destruktiivisia interferenssejä toistensa kanssa josta mallinnuksesta syntyy myös eräänlainen monimaailma mahdollisten havaitsijoiden/vuorovaikuttajien kannalta koska destruktiivinen interferenssi on vuorovaikutuksen puuttumista suhteessa havaitsijaan niin sellaiset kokonaisuudet ovat havaitsijan kannalta olemattomia ja vastaavastai vain konstuktiivisen havainto/vuorovaikutusinterferenssin tuottamat ilmiöt kuuluvat samaan maailmaan kulloinkin ko. havaitsijan kanssa ja kannalta.

        Koska erilaisilla ja myös samanlajisilla havaitsijoilla voi myös yksilötasolla olla erilaisia ominaisuuksia ja kykyjä niin käytännössä näin voi syntyä toisiinsa nähden rinakkaisia todellisuuksia havaitsijoille eli monimaailmojen kokonaisuus mikä ilmenee jos suhteessa siihen miten yksilöt kykenevät hahmottaamaan kokonaisuuksia joihin ovat perehtyneet verrattuna maallikkoihin eli huomataan eri asioita samasta kokonaisuudesta. Tässä mielessä myös esim. eri tieteenalojen ja ammattien asiantuntijat tavallaan elävät erilaisissa maailmoissa jotka eivät ole yhteismitallisia kaikilta osin.
        ...
        Kritiikkini kohdistuu lähinnä sellaiseen mekaaniseen ja kaavamaiseen monimaailmatulkintaan jossa maailmoja monistuu jatkuvasti vain sen takia että halutaan takertua universaalin determinismin ideaan.

        Mielestäni determinismi alkaa vaikuttaa vasta atomien tasolla ja on voimakkaammillaan molekyylien tasolla koska niiden paikka ja liike voidaan määritellä yksiselitteisesti ja ne ovat myös yksilöitävissä olevia entiteeettejä toisin kuin alkeishiukkasten ja kenttien tasot vaikka identiteetin määritelmä lakkaa toimimasta jos ajan eli muutokset ottaa huomioon eli perinteinen logiikka (A=A,A=B,B=C.,A=C jne. ) romahtaa jos ajan ottaa huomioon ja se toimii vain fiktiivisten kielellisten käsitteiden tasolla mistä on seurauksia myös matematiikan soveltamisen suhteen koska se logiikka että matematiikka ovat ihmisten kehittämiä loogisia konstruktioita jotka eivät sellaisenaan kerro mitään todellisuudesta vaan korkeintaan siitä miten yritämme hahmottaa sitä. ( "Jos vasara on ainoa työkalu niin kaikki mahdollinen alkaa näyttää naulamaiselta.")

        Se determinismi on myös sekä välttämätön että hyödyllinen aineen ominaisuus eliöiden ja varsinkin tietoisten ihmisten kaltaisten eliöiden tasolla koska se mahdollistaa deterministisesti käyttäytyvän aineen soveltamisen työkaluina ja tekniikkana mutta se ei tarkoita sitä että eliöt ja varsinkaan tietoiset oliot olisivat samalla tavalla niin tiukasti deterministisesti käyttäytyviä kuin se molekyylitason inertti aine. Tavallaan eliöiden kehot ovat yksilöllistymiseen pyrkivien tietoisten olioiden käyttövälineitä.

        Teorioiden ennusteiden ongelmaa olen käsitellyt muissa yhteyksissä ja samoin matematiikan ja logiikan suhdetta todellisuuteen joten ei niistä enempää.

        B

        Kuulostaa, että sekoitat nyt fysiikkaa ja jonkinlaista filosofista idealistista tai dualistista maailmankuvaa. Noiden filosofisten näkökulmien ongelma on, että ne perustuvat vain verbaaliseen kielelliseen ilmaisuun. Ne eivät ennusta mitään eikä niitä voida käyttää mihinkään hyödylliseen. Monia vastaavia ajatuksia ei voi todistaa oikeaksi sen enempää kuin vääräksikään, vaikkakin nykyinen neurologian tutkimus osoittaa jo aika lailla aukottomasti, että tietoisuus on neuroverkon prosessien tuottama taso - eikä toisinpäin.

        Tämä on kuitenkin ihan eri ongelma kuin kysymys ajan luonteesta.


      • Anonyymi
        Anonyymi kirjoitti:

        "Monimaailma tulkinta ei ole mitenkään suosittu asiaan perehtymättömien keskuudessa… "

        Riippuu aika paljon siitä mitä sillä monimaailmalla oikeasti tarkoitetaan. Jos se monimaailma koskee lähinnä vain todellisuuden mikrotasoa ja lakkaa käytännön tasolla vaikuttamasta esim. dekoherenssin ja karkeistumisen seurauksena atomeja suuremmassa mittakaavassa niin se voi vaikuttaa ihan ok tulkinnalta muiden kvanttifysiikan tason tulkintojen ohella jotka mielestäni ovat kaikki jollain tavalla virheellisiä liian kapean näkökulman takia.

        Käsittääkseni on myös sellaisia monimaailman versioita joissa ne "maailmat" voivat vuorovaikuttaa keskenään ja sellaisia joissa se mahdollisuus ei ole sallittu.

        Itse olen sillä kannalla että se alkup. ja perimmäinen todellisuus liittyy jollain tavalla epälokaaliin prototietoisuuteen josta vähitellen alkoi erilaistumaan ja samalla ehdollistumaan sellainen todellisuus joka ilmenee atomeina, molekyinä ja makrokappaleina determinismin samalla kasvaessa kun erilaistuminen edistyy.

        Jos todellisuuden hahmottaa ensisijassa aaltojen ja kenttien kautta niin se perimmäinen todellisuus on epälokaali kokonaisuus jossa ne aallot ja kentät muodostavat joko konstruktiivisia tai destruktiivisia interferenssejä toistensa kanssa josta mallinnuksesta syntyy myös eräänlainen monimaailma mahdollisten havaitsijoiden/vuorovaikuttajien kannalta koska destruktiivinen interferenssi on vuorovaikutuksen puuttumista suhteessa havaitsijaan niin sellaiset kokonaisuudet ovat havaitsijan kannalta olemattomia ja vastaavastai vain konstuktiivisen havainto/vuorovaikutusinterferenssin tuottamat ilmiöt kuuluvat samaan maailmaan kulloinkin ko. havaitsijan kanssa ja kannalta.

        Koska erilaisilla ja myös samanlajisilla havaitsijoilla voi myös yksilötasolla olla erilaisia ominaisuuksia ja kykyjä niin käytännössä näin voi syntyä toisiinsa nähden rinakkaisia todellisuuksia havaitsijoille eli monimaailmojen kokonaisuus mikä ilmenee jos suhteessa siihen miten yksilöt kykenevät hahmottaamaan kokonaisuuksia joihin ovat perehtyneet verrattuna maallikkoihin eli huomataan eri asioita samasta kokonaisuudesta. Tässä mielessä myös esim. eri tieteenalojen ja ammattien asiantuntijat tavallaan elävät erilaisissa maailmoissa jotka eivät ole yhteismitallisia kaikilta osin.
        ...
        Kritiikkini kohdistuu lähinnä sellaiseen mekaaniseen ja kaavamaiseen monimaailmatulkintaan jossa maailmoja monistuu jatkuvasti vain sen takia että halutaan takertua universaalin determinismin ideaan.

        Mielestäni determinismi alkaa vaikuttaa vasta atomien tasolla ja on voimakkaammillaan molekyylien tasolla koska niiden paikka ja liike voidaan määritellä yksiselitteisesti ja ne ovat myös yksilöitävissä olevia entiteeettejä toisin kuin alkeishiukkasten ja kenttien tasot vaikka identiteetin määritelmä lakkaa toimimasta jos ajan eli muutokset ottaa huomioon eli perinteinen logiikka (A=A,A=B,B=C.,A=C jne. ) romahtaa jos ajan ottaa huomioon ja se toimii vain fiktiivisten kielellisten käsitteiden tasolla mistä on seurauksia myös matematiikan soveltamisen suhteen koska se logiikka että matematiikka ovat ihmisten kehittämiä loogisia konstruktioita jotka eivät sellaisenaan kerro mitään todellisuudesta vaan korkeintaan siitä miten yritämme hahmottaa sitä. ( "Jos vasara on ainoa työkalu niin kaikki mahdollinen alkaa näyttää naulamaiselta.")

        Se determinismi on myös sekä välttämätön että hyödyllinen aineen ominaisuus eliöiden ja varsinkin tietoisten ihmisten kaltaisten eliöiden tasolla koska se mahdollistaa deterministisesti käyttäytyvän aineen soveltamisen työkaluina ja tekniikkana mutta se ei tarkoita sitä että eliöt ja varsinkaan tietoiset oliot olisivat samalla tavalla niin tiukasti deterministisesti käyttäytyviä kuin se molekyylitason inertti aine. Tavallaan eliöiden kehot ovat yksilöllistymiseen pyrkivien tietoisten olioiden käyttövälineitä.

        Teorioiden ennusteiden ongelmaa olen käsitellyt muissa yhteyksissä ja samoin matematiikan ja logiikan suhdetta todellisuuteen joten ei niistä enempää.

        B

        B:
        "Riippuu aika paljon siitä mitä sillä monimaailmalla oikeasti tarkoitetaan. Jos se monimaailma koskee lähinnä vain todellisuuden mikrotasoa ja lakkaa käytännön tasolla vaikuttamasta esim. dekoherenssin ja karkeistumisen seurauksena atomeja suuremmassa mittakaavassa niin se voi vaikuttaa ihan ok tulkinnalta muiden kvanttifysiikan tason tulkintojen ohella jotka mielestäni ovat kaikki jollain tavalla virheellisiä liian kapean näkökulman takia."

        Tässä on keskusteltu siitä, että voisivatko MWI ja jokin ynseämpi QM-tulkinta, joka ei pidä sitä realistisena todellisuutena, vaan laskumenetelmänä, paiskata kättä. Periaatteessa tälläiselle on sijaa siitä syystä, että MWI:ssä kuitenkin on erillainen laskumenettely siinä, että se ei romahduta tilaa. Jolloin jotakuta ynseää ihmistä voisi kiinnostaa laskea yksittäisiä asioita samalla tavalla kuin se. Muutoin pidän sitä absurdina, että filosofiseksikin usein syytetty MWI vaatisi olevansa olemassa pienten asioiden tasolla, mutta ei sitten edellyttäisikään olevansa mikään maailmaa koskeva selitysmalli, joka olisi totta.

        Tulisi olla varovaisempi siinä ettei väitä dekoherenssin olevan filosofioita toisikseen muokkaava mylly. Tai jos väittää, niin ei sitten sekoita sitä fysiikan malliin. Fysiikassa dekoherenssi on jotain, mitä esiintyy jo 16:lla kompleksiluvulla (*). Eivätkä nämä luvut pysty karistamaan MWI-keskustelua itsestään. Siten että oikeastaan mitään QM:n tarpeellisuutta ja sen mahdollista katoamista ei voida harkita. Johtuen taas siitä, että mikään klassinen ei approksimoi samaa tilannetta, mitä ne tuottavat.

        (*) Eristettynä voi esiintyä ennen ja jälkeen -keskustelu. Muutoin varsinainen dekoherenssin etenemä tarkoittaa jonkin ympäristön vaikutusta. Samalla kun ympärisö vaikuttaa, on kuitenkin huomattavaa, että huomiomme voidaan koko ajan keskittää seuraamaan näitä 16 kompleksilukua, mikä ei ole sama kuin mitä tarkoitettaisiin sillä, että antaa kaiken olla karkeaa.

        B:
        "Käsittääkseni on myös sellaisia monimaailman versioita joissa ne "maailmat" voivat vuorovaikuttaa keskenään ja sellaisia joissa se mahdollisuus ei ole sallittu."

        Jos tarkoitetaan muutamalle kappaleelle niitä Hilbertin avaruuden suuntia joiden välillä on korrelaatio, niin mikä tahansa voi tulla ja vuorovaikuttaa nykyisistä suunnista ristiin toisiin tiloihin luoden esim. korrelaatiot eri tiloihin tai tehdä niistä jälleen kvanttikietoumia. Yhden tällaisen kappaleen dekoheroinut ympäristö on kuitenkin se, mikä on myös maailmaa MWI:ssä, ja se on siis usean kappaleen järjestelmä. Mistä kaikista on jokin korrelaatio yhteen näihin objekteihin ja sen tulisi muuttua. Se olisi hyvin sama kuin sallisi koko maailman menettää klassisen luonteensa (hetkittäinen tila), ja tulla myös samalla hetkellä yhteen yhtä reaaliaikaista vuorovaikutusta varten (menetelmä). Tästä puhutaan videoissa myös sanomalla, että kvantti-informaatio (mm. sitä kun korrelaatio on kietoutumisena tai sitä että yksittäinen objekti oli superpositio) on levinnyt muualle aineeseen.

        Samanlaiseen asiaan liittyvää: Kun on olemassa yksittäinen hiukkanen, niin sellaiselle on ominaista, että sen Hilbertin avaruuden suuntien välillä ei ole vuorovaikutusta, mistä seuraa, että mainitut ns. historiat pelkässä superpositiossa, eivät itsestään katkeile ja paukahtele. Epästaabiilitkin hiukkaset ovat esitettävissä kvanttityhjiön avulla, jossa vuorovaikutus kytkee nämä annetut suunnat toiseen joukkoon toisten hiukkasten suuntia.

        1


      • Anonyymi
        Anonyymi kirjoitti:

        B:
        "Riippuu aika paljon siitä mitä sillä monimaailmalla oikeasti tarkoitetaan. Jos se monimaailma koskee lähinnä vain todellisuuden mikrotasoa ja lakkaa käytännön tasolla vaikuttamasta esim. dekoherenssin ja karkeistumisen seurauksena atomeja suuremmassa mittakaavassa niin se voi vaikuttaa ihan ok tulkinnalta muiden kvanttifysiikan tason tulkintojen ohella jotka mielestäni ovat kaikki jollain tavalla virheellisiä liian kapean näkökulman takia."

        Tässä on keskusteltu siitä, että voisivatko MWI ja jokin ynseämpi QM-tulkinta, joka ei pidä sitä realistisena todellisuutena, vaan laskumenetelmänä, paiskata kättä. Periaatteessa tälläiselle on sijaa siitä syystä, että MWI:ssä kuitenkin on erillainen laskumenettely siinä, että se ei romahduta tilaa. Jolloin jotakuta ynseää ihmistä voisi kiinnostaa laskea yksittäisiä asioita samalla tavalla kuin se. Muutoin pidän sitä absurdina, että filosofiseksikin usein syytetty MWI vaatisi olevansa olemassa pienten asioiden tasolla, mutta ei sitten edellyttäisikään olevansa mikään maailmaa koskeva selitysmalli, joka olisi totta.

        Tulisi olla varovaisempi siinä ettei väitä dekoherenssin olevan filosofioita toisikseen muokkaava mylly. Tai jos väittää, niin ei sitten sekoita sitä fysiikan malliin. Fysiikassa dekoherenssi on jotain, mitä esiintyy jo 16:lla kompleksiluvulla (*). Eivätkä nämä luvut pysty karistamaan MWI-keskustelua itsestään. Siten että oikeastaan mitään QM:n tarpeellisuutta ja sen mahdollista katoamista ei voida harkita. Johtuen taas siitä, että mikään klassinen ei approksimoi samaa tilannetta, mitä ne tuottavat.

        (*) Eristettynä voi esiintyä ennen ja jälkeen -keskustelu. Muutoin varsinainen dekoherenssin etenemä tarkoittaa jonkin ympäristön vaikutusta. Samalla kun ympärisö vaikuttaa, on kuitenkin huomattavaa, että huomiomme voidaan koko ajan keskittää seuraamaan näitä 16 kompleksilukua, mikä ei ole sama kuin mitä tarkoitettaisiin sillä, että antaa kaiken olla karkeaa.

        B:
        "Käsittääkseni on myös sellaisia monimaailman versioita joissa ne "maailmat" voivat vuorovaikuttaa keskenään ja sellaisia joissa se mahdollisuus ei ole sallittu."

        Jos tarkoitetaan muutamalle kappaleelle niitä Hilbertin avaruuden suuntia joiden välillä on korrelaatio, niin mikä tahansa voi tulla ja vuorovaikuttaa nykyisistä suunnista ristiin toisiin tiloihin luoden esim. korrelaatiot eri tiloihin tai tehdä niistä jälleen kvanttikietoumia. Yhden tällaisen kappaleen dekoheroinut ympäristö on kuitenkin se, mikä on myös maailmaa MWI:ssä, ja se on siis usean kappaleen järjestelmä. Mistä kaikista on jokin korrelaatio yhteen näihin objekteihin ja sen tulisi muuttua. Se olisi hyvin sama kuin sallisi koko maailman menettää klassisen luonteensa (hetkittäinen tila), ja tulla myös samalla hetkellä yhteen yhtä reaaliaikaista vuorovaikutusta varten (menetelmä). Tästä puhutaan videoissa myös sanomalla, että kvantti-informaatio (mm. sitä kun korrelaatio on kietoutumisena tai sitä että yksittäinen objekti oli superpositio) on levinnyt muualle aineeseen.

        Samanlaiseen asiaan liittyvää: Kun on olemassa yksittäinen hiukkanen, niin sellaiselle on ominaista, että sen Hilbertin avaruuden suuntien välillä ei ole vuorovaikutusta, mistä seuraa, että mainitut ns. historiat pelkässä superpositiossa, eivät itsestään katkeile ja paukahtele. Epästaabiilitkin hiukkaset ovat esitettävissä kvanttityhjiön avulla, jossa vuorovaikutus kytkee nämä annetut suunnat toiseen joukkoon toisten hiukkasten suuntia.

        1

        B:
        "Jos todellisuuden hahmottaa ensisijassa aaltojen ja kenttien kautta niin se perimmäinen todellisuus on epälokaali kokonaisuus jossa ne aallot ja kentät muodostavat joko konstruktiivisia tai destruktiivisia interferenssejä toistensa kanssa josta mallinnuksesta syntyy myös eräänlainen monimaailma mahdollisten havaitsijoiden/vuorovaikuttajien kannalta koska destruktiivinen interferenssi on vuorovaikutuksen puuttumista suhteessa havaitsijaan niin sellaiset kokonaisuudet ovat havaitsijan kannalta olemattomia ja vastaavastai vain konstuktiivisen havainto/vuorovaikutusinterferenssin tuottamat ilmiöt kuuluvat samaan maailmaan kulloinkin ko. havaitsijan kanssa ja kannalta."

        Aaltopari jolla on konstruktiivista interferenssiä, on yleensä myös destruktiivista interferenssiä samalla.

        1. Konstruktiivista interferenssiä ei ole olemassa, ellei sitä havaitse.
        2. Jos havaitsee destruktiivista interferenssiä, sitä on olemassa.
        3. Jos havaitsee -interferenssiä, havaitsee aallon.
        4. Maailma on sitä mitä havaitsee.
        5. (MWI-tyylinen lause) Jos sitä havaitsee, on korreloitunut sen kanssa.
        Eli. Jos havaitsee -interferenssiä, on korreloitunut aallon kanssa, ja maailmassa on aalto.

        Tosin QM-aaltoa ei voi havaita yhdellä mittauksella, mutta QM-kokeista voi tehdä tuon päätelmän, että aaltoja on ja oli olemassa jokainen kerta.

        Korreloituminen aallon kanssa on tavalliselta nimeltään pikemminkin nollakorrelaatio. Ja maailmojen määrä myös nolla.

        Aaltojen katoaminen eli ns. muuttuminen hiukkasmaisiksi on vielä asia erikseen ja tulee vasta tämän jälkeen. Sinun tekstissäsi on periaatteessa haluttu puhua lopussa tapauksesta, missä QM-aaltoa ei voi nähdä enää, jos sen esim. mittaa toiseen kertaan. Aloitit kappaleen kuitenkin sanomalla, että ei olisi yhtään mielekästä ajatella mitään muuta kuin aaltoja. Mistä voisi päätellä, että sinun on pakko hyväksyä MWI ja aaltojen jättämät hiukkasmaiset korrelaatiot ajatellaksesi noin.

        QM on objektiivinen teoria siten, että siinä jonkun muun tekemä havaitseminen jostain asiasta (joka tulee muuttamaan tätä toista kuten sanotaan) muuttaa sitä myös minkä tahansa kannalta. Lisäksi interferenssi on asia, missä aalto (joka on myös aallot, koska 'aallot' viittaa siihen, että se on superpositiossa )ei tarvita ketään muuta. Kun objekti muuttuu itse tekemällä interferenssejä itsensä kanssa, tämä tulee olemaan vaikuttava tekijä siinä, mitä siitä voitaisiin havaita. Jos esim. destruktiivinen interferenssi on täyttä, niin todennäköisyys sellaisessa tilassa mitatuksi tulemiselle on nolla. Korrelaatiollakin pitää olla todennäköisyyttä ollakseen korrelaatio, joten tätä voi sanoa maailmattomaksi Hilbertin avaruuden suunnaksi.

        Jos ajattelee kaksoisrakokokeen takalevyä, niin destruktiivinen interferenssi on tapahtunut jo ennen sitä. Ja alueet, missä sitä on ollut, ovat alueita, joilla ei edes ole mahdollisuutta vuorovaikuttaa minkään kanssa. Tai tarkemmin sanottuna vuorovaikutus Hilbertin avaruuden suunnan kanssa, jolla on todennäköisyys jo alussa nolla, ei tuota yhtään mitään siitä eteenpäin siirrettynä todennäköisyysamplitudina. Tämä on pikemminkin osoitus siitä, että jokin on teoriassa (*) lokaalista, koska destruktiivisille avaruuden alueille ei anneta amplitudia sieltä, missä sitä on.

        (*) Varsinkin levyn ja aallon vuorovaikutuksissa. Jos jokin tilanteen malli olisi rakennettu vaiheittain, se olisi saattanut esim. vaatia jostain syystä alusta alkaen, että kaksoisraon jälkeisessä aallossa ei voi olla olematta destruktiivisen interferenssin alueita. Tämän jälkeen, jos tästä päätelmästä ei suostu joustamaan, niin on pakollista sanoa, että vuorovaikutus levyllä on lokaalia, koska levy ei muuta aaltoa tai sen aiheuttamaa seurausta eli merkkejä levyllä takaisin tasaisesti levitetyksi aalloksi. Onko destruktiivisuus pakollista, voidaan tarkistaa ottamalla raot joskus pois tai käyttämällä yhtä.

        B:
        "destruktiivinen interferenssi on vuorovaikutuksen puuttumista suhteessa havaitsijaan"

        Jos tämän ajattelee väärin, tästä olisi yllä seurauksena se, että materia katoaa universumista kaksoisrakokoetta tehdessä. Sinun tulee käsitellä ajatuksiasi matematisoituina ja epäkonkreettisina käsitteinä, kuten 'Hilbertin avaruuden suunta', jos haluat sanoa, että on jokin X, mikä lakkauttaa kaiken vuorovaikutuksensa muihin. Tai edes osaan heistä. Koska jos haluat että aine on täysin subjektiivinen kokemus eikä kenellekään sama (nämä 'kenet' eivät vuorovaiktuta keskenään sen vertaa, että muodostaisivat yhden maailman?), niin siinäkin postuloidaan monia äskeisenkaltaisia objekteja.

        ...

        2


      • Anonyymi
        Anonyymi kirjoitti:

        B:
        "Jos todellisuuden hahmottaa ensisijassa aaltojen ja kenttien kautta niin se perimmäinen todellisuus on epälokaali kokonaisuus jossa ne aallot ja kentät muodostavat joko konstruktiivisia tai destruktiivisia interferenssejä toistensa kanssa josta mallinnuksesta syntyy myös eräänlainen monimaailma mahdollisten havaitsijoiden/vuorovaikuttajien kannalta koska destruktiivinen interferenssi on vuorovaikutuksen puuttumista suhteessa havaitsijaan niin sellaiset kokonaisuudet ovat havaitsijan kannalta olemattomia ja vastaavastai vain konstuktiivisen havainto/vuorovaikutusinterferenssin tuottamat ilmiöt kuuluvat samaan maailmaan kulloinkin ko. havaitsijan kanssa ja kannalta."

        Aaltopari jolla on konstruktiivista interferenssiä, on yleensä myös destruktiivista interferenssiä samalla.

        1. Konstruktiivista interferenssiä ei ole olemassa, ellei sitä havaitse.
        2. Jos havaitsee destruktiivista interferenssiä, sitä on olemassa.
        3. Jos havaitsee -interferenssiä, havaitsee aallon.
        4. Maailma on sitä mitä havaitsee.
        5. (MWI-tyylinen lause) Jos sitä havaitsee, on korreloitunut sen kanssa.
        Eli. Jos havaitsee -interferenssiä, on korreloitunut aallon kanssa, ja maailmassa on aalto.

        Tosin QM-aaltoa ei voi havaita yhdellä mittauksella, mutta QM-kokeista voi tehdä tuon päätelmän, että aaltoja on ja oli olemassa jokainen kerta.

        Korreloituminen aallon kanssa on tavalliselta nimeltään pikemminkin nollakorrelaatio. Ja maailmojen määrä myös nolla.

        Aaltojen katoaminen eli ns. muuttuminen hiukkasmaisiksi on vielä asia erikseen ja tulee vasta tämän jälkeen. Sinun tekstissäsi on periaatteessa haluttu puhua lopussa tapauksesta, missä QM-aaltoa ei voi nähdä enää, jos sen esim. mittaa toiseen kertaan. Aloitit kappaleen kuitenkin sanomalla, että ei olisi yhtään mielekästä ajatella mitään muuta kuin aaltoja. Mistä voisi päätellä, että sinun on pakko hyväksyä MWI ja aaltojen jättämät hiukkasmaiset korrelaatiot ajatellaksesi noin.

        QM on objektiivinen teoria siten, että siinä jonkun muun tekemä havaitseminen jostain asiasta (joka tulee muuttamaan tätä toista kuten sanotaan) muuttaa sitä myös minkä tahansa kannalta. Lisäksi interferenssi on asia, missä aalto (joka on myös aallot, koska 'aallot' viittaa siihen, että se on superpositiossa )ei tarvita ketään muuta. Kun objekti muuttuu itse tekemällä interferenssejä itsensä kanssa, tämä tulee olemaan vaikuttava tekijä siinä, mitä siitä voitaisiin havaita. Jos esim. destruktiivinen interferenssi on täyttä, niin todennäköisyys sellaisessa tilassa mitatuksi tulemiselle on nolla. Korrelaatiollakin pitää olla todennäköisyyttä ollakseen korrelaatio, joten tätä voi sanoa maailmattomaksi Hilbertin avaruuden suunnaksi.

        Jos ajattelee kaksoisrakokokeen takalevyä, niin destruktiivinen interferenssi on tapahtunut jo ennen sitä. Ja alueet, missä sitä on ollut, ovat alueita, joilla ei edes ole mahdollisuutta vuorovaikuttaa minkään kanssa. Tai tarkemmin sanottuna vuorovaikutus Hilbertin avaruuden suunnan kanssa, jolla on todennäköisyys jo alussa nolla, ei tuota yhtään mitään siitä eteenpäin siirrettynä todennäköisyysamplitudina. Tämä on pikemminkin osoitus siitä, että jokin on teoriassa (*) lokaalista, koska destruktiivisille avaruuden alueille ei anneta amplitudia sieltä, missä sitä on.

        (*) Varsinkin levyn ja aallon vuorovaikutuksissa. Jos jokin tilanteen malli olisi rakennettu vaiheittain, se olisi saattanut esim. vaatia jostain syystä alusta alkaen, että kaksoisraon jälkeisessä aallossa ei voi olla olematta destruktiivisen interferenssin alueita. Tämän jälkeen, jos tästä päätelmästä ei suostu joustamaan, niin on pakollista sanoa, että vuorovaikutus levyllä on lokaalia, koska levy ei muuta aaltoa tai sen aiheuttamaa seurausta eli merkkejä levyllä takaisin tasaisesti levitetyksi aalloksi. Onko destruktiivisuus pakollista, voidaan tarkistaa ottamalla raot joskus pois tai käyttämällä yhtä.

        B:
        "destruktiivinen interferenssi on vuorovaikutuksen puuttumista suhteessa havaitsijaan"

        Jos tämän ajattelee väärin, tästä olisi yllä seurauksena se, että materia katoaa universumista kaksoisrakokoetta tehdessä. Sinun tulee käsitellä ajatuksiasi matematisoituina ja epäkonkreettisina käsitteinä, kuten 'Hilbertin avaruuden suunta', jos haluat sanoa, että on jokin X, mikä lakkauttaa kaiken vuorovaikutuksensa muihin. Tai edes osaan heistä. Koska jos haluat että aine on täysin subjektiivinen kokemus eikä kenellekään sama (nämä 'kenet' eivät vuorovaiktuta keskenään sen vertaa, että muodostaisivat yhden maailman?), niin siinäkin postuloidaan monia äskeisenkaltaisia objekteja.

        ...

        2

        Interferenssit joita on kaksoisrakokokeessa koko levyn alueella, ovat jotain joita on monta. Keskittymällä niihin ja pitämällä niitä lauseenjäseninä olet jättänyt kertomatta tähän mennessä sen, että mitä mittauksessa tapahtuu, eli miksi usean konstruktiivisen interferenssin ollessa läsnä per yksi saapuva aalto, niistä muodostuu pian vain yksi piste levyllä. Toki MWI:lläkin on vain täysin olematon tarina mittauksesta, mutta ei kai tarkoitus ole pitää sitä riman asettajana?

        B:
        "Kritiikkini kohdistuu lähinnä sellaiseen mekaaniseen ja kaavamaiseen monimaailmatulkintaan jossa maailmoja monistuu jatkuvasti vain sen takia että halutaan takertua universaalin determinismin ideaan."

        Voisit kohdistaa kritiikin myös itseesi, koska se mitä sanoit jonkinlaisesta subjektiivisesta todellisuudesta, ei ole vielä mitenkään epädeterminististä siitä mainittujen tietojen perusteella. Esim. keksimääsi todellisuutta ei voi verrata QM-satunnaisuuteen, koska tiedetään, että QM-satunnaisuudessa tai siinä maailmassa missä on QM-kokeita ja ihmisiä keskustelemassa niistä, maailma on heille objektiivinen. Joten tulisi ajatella, että kaikki keksinnössäsi on vielä määrittelemättä.

        B:
        "Mielestäni determinismi alkaa vaikuttaa vasta atomien tasolla ja on voimakkaammillaan molekyylien tasolla koska niiden paikka ja liike voidaan määritellä yksiselitteisesti ja ne ovat myös yksilöitävissä olevia entiteeettejä toisin kuin alkeishiukkasten ja kenttien tasot"

        Determinismin käyttöä ei ole mitään syytä vaihtaa tuossa kohtaa, koska tästä maailmasta tiedetään, että atomit voivat olla samassa QM-liikeyhtälön liikkeessä kuin elektroni, ja sen lisäksi suurin osa molekyyleistä voi olla samassa QM-liikkeessä kuin elektroni.

        B:
        " identiteetin määritelmä lakkaa toimimasta jos ajan eli muutokset ottaa huomioon eli perinteinen logiikka (A=A,A=B,B=C.,A=C jne. ) romahtaa jos ajan ottaa huomioon"

        QM:ssä on niin, että voidaan konstruoida tapaus, missä ei ole tietoa siitä, mikä monesta hiukkasesta on mikin hiukkanen, mutta tämä tieto on poissa myös jos aikaa ei ole. Olisit myös ristiriidassa klassisuuden kanssa. Miksi klassiset yritykset tehdä elektroni tai atomi ovat identifioitavissa olevia elektroneja ja atomeja, vaikka aika juoksee?

        Kun on olemassa isoja molekyylejä, myös molekyyliä voidaan erehtyä pitämään toisena identtisenä molekyylinä, mutta tässä on tehty karkeistus ja pidetty molekyyliä sen tason objektina että sillä tasolla myös ollaan. Tosiasiassa luonnossa ei ole mitään tasoja vaan pelkät elektronit. Tällöin ei ole mitään aineen kokoonpanoa, missä elektronit eivät olisi identiteetittömiä. Identiteetittömyys ei reagoi kokeissa lainkaan samalla tavalla kuin esim. superpositiot ja aineen klassistuminen. Tämän takia monikin kaasu pysyy näkyvällä tavalla FD- tai BE-kaasuna. Minkä lisäksi se kaasu, miksi ne muuttuisivat, on jotain, mikä on myös identiteettitömyyden mukaista. Eli klassisessa aineessa mikään ominaisuus ei näytä katoavan vaan siinä aineessa ei pystytä näkemään sitä, että pystyttäisiinkö näkemään, mikä hiukkanen on mikäkin. Jos otetaan kaksi suurta kiinteää objektia, jotka yrittävät olla sama koko rakenteeltaan ja sen sijoittelulta, niin silloin QM-identiteetittömyys karkealle esineelle toimii täysin QM:n mukaan, mutta mitä efektiä tästä seuraisi, on täysin olematonta.

        Oikeastaan äskeisessä ei ole mahdollista olettaa, että universumi sisältäisi mitään elektronia suurempaa kiinteää kappaletta, jos se ei olisi muodostanut sitä sillä, mitä identiteettittömyydestä seuraa. Eli fermionista vuorovaikutusta.

        Kaksi identtistä hiukkasta ei pysty tuhoamaan logiikkaa sen sisältä käsin tai edes itselleen, koska se mitä näistä hiukkasista väitetään identiteetti tiedon puuttuessa on muotoa, mikä on 'hiukkasen1' käsitettynä 'sijaintina'. Koska hiukkasen määritelmä ei ole sen sijainti, niin hiukkanen ei muutu miksikään vaikka sanoo, että sen sijainti on mitä tahansa. Tai toisin sanoen, hiukkasesta ei esitetä mitään väitettä, kun sen sijainnista väitetään jotain, jolloin hiukkanen1=hiukkanen1 on väite johon ei kohdistu painetta.

        Ehkä oleellisempaa on, että rikomatta mitään logiikkaa, pystyy rakentamaan sen matematiikan, missä on identiteetittömät hiukkaset. Eikä senkään jälkeen ole kyseessä ristiriita.

        3


    • Anonyymi

      Vanhastaan tiedetty, että aika on rahaa.

    • Anonyymi

      "Aiheeseen liittyy nähdäkseni myös kysymys monimaailmoista. Vaikuttaisi, että systeemin sisällä on olemassa lukuisia rinnakkaisia mahdollisia historioita ja systeemin ulkopuolinen voi mallintaa sitä oikein vain laskemalla kaikkien mahdollisten historioiden yhteisvaikutuksen."

      Ulkopuolinen näkökulma: tämän määritelmä on ensimmäisissä linkeissä olla kietoutumatta systeemin kanssa ja olla muodostamatta kelloa systeemin osasta. Minkä seurauksena jos edelleen ajattelee olevan kyseessä näkökulma, ja yrittää käyttää sitä oletuksellisesti näkökulmana, missä tikittää absoluuttinen aika, niin silloin systeemi ei näytä muuttuvan ajassa.

      Ei ole määritelty sellaista ulkopuolista näkökulmaa, joka olisi monien maailmojen ulkopuolella nähden juuri ne kaikki. Jos ei ole väliä, tullanko löytämään mitään ajatusten seurauksena, niin ryhtyminen monen maailman näkökulmaksi voi käydä sanomalla, että on siinä näkökulmassa kuin äskenkin, mutta sen sijaan, että on tapahtuma kuin mittaus, jonka seurauksena on yksi maailma, niin niitä onkin monta. Tässä kävi kuitenkin niin, että sanottiin systeemin olevan alussa jotain ennen tapahtumaa, ja jotain muuta sen jälkeen. On siis todennäköisesti epäonnistuttu muodostamaan näkökulmaa, jossa aikaa ei olisi. Eikä ajatus, mitä tässä käydään läpi, tule silloin koskemaan ajan uutta luonnetta.

      Ja tietenkään mikään, mitä hän näkee, ei saa olla 'historia'.

      Ehkä ulkopuolisen näkemän muuttumattoman tilan pitää aina olla täysi kvanttikoherenssi, missä tapauksessa hän ei koskaan näe dekoheroituneita maailmoja. Jolloin hänestä ei olisi mitään syytä tehdä tulkintoja asiasta, 'mitä dekoherenssin jälkeen on olemassa?' eli yksi maailma vai monta.

      ...

      Systeemin sisällä olevan näkökulman puolestaan täytyy olla lähellä MWI-tulkintaa, koska systeemi jaetaan kahteen osaan ja sen jälkeen on olemassa tietty tapa sanoa, miten yksi osa näyttäytyy toiselle osalle. Tämän tavan kirjoittaa keksi ensin MWI, mutta sitä käytetään myös RQM-tulkinnassa, jonka mielestä kaikkia myöhemmin yhdessä nähtäviä asioita ei pidetä maailmana. Uuden kellon teoriassa riittää ajatella, että kellon muuttuminen (joka ei kenenkään mielestä ole muuttumista ajassa) on yksittäinen kellon ominaistila kerrallaan eikä edes superpositio. Muutoin voitaisiin mm. kysyä, että onko aika sitä, että mitataan tämä kello, ja jos se mitataan monta kertaa, niin miksi tulokset olisivat suuruusjärjestyksessä. Vaikka tulkinnan tarve puuttuu kuin täysin, niin silti tämä sisäinen näkökulma on se, missä MWI:tä voisi käyttää lisää, kunhan asioita tapahtuisi enemmän.

      Kokeissa mitä asiasta tehdään, niin mittaus on aina vahvemmin mukana kuin se on mallissa oikeasti. Koska sekä ulkoiset että sisäiset tutkijat ottavat myös selvää siitä, mitä kello ja systeemi ovat. Ja se mitä mitataan voi olla ylimääräisellä tavalla monimutkaisempi kuin peruskello.

      "ulkopuolinen voi mallintaa sitä oikein vain laskemalla kaikkien mahdollisten historioiden yhteisvaikutuksen."

      Tämä ei koske pelkkää MWI:tä, vaan sanomasi koskee myös superpositioiden osatilojen historioita, jotka ovat lineaarisesti erillisiä ennen tapahtumaa, missä tulkinta päättää, mitä seuraavaksi tehdään. Tämä on juuri kaikkein oleellisinta silloin, jos olet systeemissä sisällä, ja sinulla on kello ja koko muu maailmankaikkeus.

      ...

      B:
      "Jos tuon kvanttifysiikan monimaailmatulkinnan korvaa sillä idealla että potentiaalisuus on yhtä todellista kuin empiirisesti koettu aineellinen todellisuus niin kokonaisuus on huomattavasti selkeämpi."

      Jos potentiaalisuudella ei ole mitään tekemistä energian kanssa, vaan se kuvaa vaihtoehtojen määrää superposition jälkeen mitatussa tilassa, niin MWI nimenomaan sanoo, että vaihtoehdot ovat yhtä todellisia kuin koettu tulos. Etkä siis lisännyt MWI:hin mitään tällä lauseella.

      B:
      "Potentiaalisuus on ainakin makrotasolla aina suurempi kuin se mikä voi fysikaalisena todellisuutena aktualisoitua. "

      Tämä on loogisesti sama kuin neuvo, että jos joku puhuu potentiaalisuudesta, niin pidä varasi sen suhteen, että jotkut asiat eivät kuitenkaan koskaan toteudu.

      (Kirjoitit 'ainakin':) Ilman MWI:tä makrotaso on makrotaso, koska sillä ainoa potentiaalisuus on se, mitä se juuri nyt on, mutta juuri siinä mielessä, että nimenomaan tiedetään, että sillä ei ole mitään muuta vaihtoehtoa kuin vain yksi. Eli nimenomaan makrotaso on vaihtoehtojen määrästä riisuttu eikä toisin päin. Ja näin sanoit itsekkin myöhemmin.

      1

      • Anonyymi

        B:
        "Esim. fysiikassa ja myös ihan arjen tason kokemuksessa tunnetaan potentiaalinergian käsite joka tarkoittaa sitä että johonkin fysikaaliseen kohteeseen on varastoitunut energiaa (esim. viritetty jousi) jonka aktualisoitumisen tapa ja suunta on epävarmaa siihen asti kunnes se varastoitunut energia pääsee purkautumaan."

        Arjen taso on Newtoninen, ja esim. sillä tavalla se sisältää jo ennusteen siitä henkilöstä, joka eräänä päivänä tulee ja saa jousen näpeilleen tiettyyn sormeen. Tässä energia on täysin turha lisäkäsite, koska voisi olla olemassa esim. täysin nollaenergiaa oleva muuttujan vaihteluväli, kuten magneetin suunta (kentän suuntana). Voitaisiin arjellisesti ajatella, että kellään ei ole vielä käsitystäkään, minkä suuntainen magneetti on huomenna, ja että silloin jokin suunnista purkautuu tiedettäväksemme.

        B:
        "Schrödingerin aaltiyhtälö ei liity fysikaalisiin aaltoihin vaan aaltomaisesti käyttäytyviin todennäköisyysaaaltoihin joiden mukaan yritetään määritellä hiukkasten liikkeiden tulevaisuus tilastollisella tasolla jolloin kyseessä on nimenomaan epävarmuus joka seurauksena hiukkasilla on enemmän apausasteita toisin kuin esim. kiinteillä biljadipalloilla jotka käyttäytyvät suhteellisen deterministisesti vaikka kuluvat käytössä ajan kuluessa."

        Sanoit ensin, että potentiaalisuudet eli todennäköisyydet ovat yhtä todellisia kuin kaikki mikä on varmaa. Tarkoititko siis, että edes varma ja tapahtunut todellisuus ei ole fysikaalista?

        Mieti myös, että onko hiukkaselle annettu vapaus (kyseessä ei ole edes Hilbertin avaruuden vapausaste vaan kyseessä on yksittäisten tilojen täyttäminen niillä) fysikaalista vai ei. Ja että jos joku antaa sitä, niin miksi se jokin eli epävarmuus ei ole fysikaalinen? Tosiasiassa 'antaminen/seuraus' on turha vaihe, joka ilmestyy tekstiin ja päähän, kun tosiasiassa tässä vain siirrytään saman asian esittämisestä toiseen tautologiaan.

        B:
        "Varmuus ja determinismi pätee oikeastaan vain kiinteiden jäykkien kappaleiden tasolla ainakin jossain määrin..."

        Kumpikin noista pätee myös ideaalisessa nesteessä (tätä on olemassa yhtä paljon kuin ideaalista jäykkyyttä), jota aallotutetaan edes takaisin tai ylös ja alas, mistä tulee varmaa ja determinististä. Minkä lisäksi nämä pätevät muissakin aalloissa.

        Oikeastaan kappale vs. neste tai kaasu on esitetty tällä tavalla vain niiden kanssa toimineiden fyysikoiden sanomisissa, kun he jäävät tarkoituksella epävarmoiksi esim. kaasun suhteen. Voisit käyttää mielummin omaa ajatteluasi joskus kuin seurata heitä.

        Kaksi jäykkää kappaletta pystyy muodostamaan epäjäykän kappaleen. Mutta pystytkö muodostamaan kahdesta jäykästä kappaleesta epävarman asian? Tässä joko ei pysty jakamaan maailmaa jäykkiin kappaleisiin tai sitten varmuuden ja determinismin toimialue on suurempi kuin luetteloit.

        B:
        "...mutta koska luonnontieteissä on perinteisesti pyritti mahdollisimman suureen varmuuteen ja tarkkuuteen niin tietyssä mielessä monimaailmatulkinta (ossa kaikki mahdollinen toteutuu omassa deterministisessä maailmassaan) voi tuntua joidenkin mielestä houkuttelevalta vaihtoehdolta jos tutkijoiden epävarmuuden sietokyky on heikko ja ainoa työkalu on matemaattinen hahmotus determinististen kaavojen avulla."

        MWI:ssä ei voi muodostaa matemaattista alijärjestelmää tai tilannetta, jossa on yksi maailma otettuna useista, ja joka toimii deterministisesti kaavan mukaan. Siksi se on Many-WI eikä One-WI. Työkalujen maininta on väärin siten, että niitä ei ole lisätty, vaan MWI riisuu esim. Kööpenhaminan tulkinnan työkaluista, mutta samalla lisäten jäljellä olevien työkalujen sovelluskertoja. Tai voi sanoa myös entiteettien lukumäärän kasvaneen (missä entiteetti ei ole uudenlainen tai ulkopuolinen; eikä entiteetti ole myöskään maailma).

        B:
        "Monimaailmamalli käytännössä tuottaisi valtavan määrän versumeita jokaisen hiukkastaon vuorovaikutuksen seurauksena ja jo pelkästään logiikan Reductio ad absurdum periaatteen mukaan pitäisi hylätä täysin järjettömänä ja epärealistisena tulkintana."

        Jos esimerkkitodellisuuksia saataisiin ehdottaa vapaasti, niin yhdessä, mikä tehdään MWI-pohjalta, ei ole mitään, mitä ei voisi pitää oikeana todellisuutena. Käytännössä MWI-malli on multiversumi, joka sisältää esim. 16 kompleksilukua eikä mitään muuta ole olemassa. Jos sanotaan, että jonkin tapahtuman jälkeen tässä todellisuudessa on kaksi eri maailmaa hamaan loppuun asti, niin se on silti edelleen esitettävissä 16 kompleksiluvulla. Se ei ole paljon. Lisäksi vastaväittäjä ei pysty koskaan tekemään mallia, jolla on samat ominaisuudet (mutta jota ei sanota MWI:ksi) ja joka voitaisiin esittää alle 16 kompleksiluvulla, joten seurauksena kaikki mallit olisivat väärin jos yksi on sen takia, että sen piti olla 16 tai enemmän.

        2


      • Anonyymi
        Anonyymi kirjoitti:

        B:
        "Esim. fysiikassa ja myös ihan arjen tason kokemuksessa tunnetaan potentiaalinergian käsite joka tarkoittaa sitä että johonkin fysikaaliseen kohteeseen on varastoitunut energiaa (esim. viritetty jousi) jonka aktualisoitumisen tapa ja suunta on epävarmaa siihen asti kunnes se varastoitunut energia pääsee purkautumaan."

        Arjen taso on Newtoninen, ja esim. sillä tavalla se sisältää jo ennusteen siitä henkilöstä, joka eräänä päivänä tulee ja saa jousen näpeilleen tiettyyn sormeen. Tässä energia on täysin turha lisäkäsite, koska voisi olla olemassa esim. täysin nollaenergiaa oleva muuttujan vaihteluväli, kuten magneetin suunta (kentän suuntana). Voitaisiin arjellisesti ajatella, että kellään ei ole vielä käsitystäkään, minkä suuntainen magneetti on huomenna, ja että silloin jokin suunnista purkautuu tiedettäväksemme.

        B:
        "Schrödingerin aaltiyhtälö ei liity fysikaalisiin aaltoihin vaan aaltomaisesti käyttäytyviin todennäköisyysaaaltoihin joiden mukaan yritetään määritellä hiukkasten liikkeiden tulevaisuus tilastollisella tasolla jolloin kyseessä on nimenomaan epävarmuus joka seurauksena hiukkasilla on enemmän apausasteita toisin kuin esim. kiinteillä biljadipalloilla jotka käyttäytyvät suhteellisen deterministisesti vaikka kuluvat käytössä ajan kuluessa."

        Sanoit ensin, että potentiaalisuudet eli todennäköisyydet ovat yhtä todellisia kuin kaikki mikä on varmaa. Tarkoititko siis, että edes varma ja tapahtunut todellisuus ei ole fysikaalista?

        Mieti myös, että onko hiukkaselle annettu vapaus (kyseessä ei ole edes Hilbertin avaruuden vapausaste vaan kyseessä on yksittäisten tilojen täyttäminen niillä) fysikaalista vai ei. Ja että jos joku antaa sitä, niin miksi se jokin eli epävarmuus ei ole fysikaalinen? Tosiasiassa 'antaminen/seuraus' on turha vaihe, joka ilmestyy tekstiin ja päähän, kun tosiasiassa tässä vain siirrytään saman asian esittämisestä toiseen tautologiaan.

        B:
        "Varmuus ja determinismi pätee oikeastaan vain kiinteiden jäykkien kappaleiden tasolla ainakin jossain määrin..."

        Kumpikin noista pätee myös ideaalisessa nesteessä (tätä on olemassa yhtä paljon kuin ideaalista jäykkyyttä), jota aallotutetaan edes takaisin tai ylös ja alas, mistä tulee varmaa ja determinististä. Minkä lisäksi nämä pätevät muissakin aalloissa.

        Oikeastaan kappale vs. neste tai kaasu on esitetty tällä tavalla vain niiden kanssa toimineiden fyysikoiden sanomisissa, kun he jäävät tarkoituksella epävarmoiksi esim. kaasun suhteen. Voisit käyttää mielummin omaa ajatteluasi joskus kuin seurata heitä.

        Kaksi jäykkää kappaletta pystyy muodostamaan epäjäykän kappaleen. Mutta pystytkö muodostamaan kahdesta jäykästä kappaleesta epävarman asian? Tässä joko ei pysty jakamaan maailmaa jäykkiin kappaleisiin tai sitten varmuuden ja determinismin toimialue on suurempi kuin luetteloit.

        B:
        "...mutta koska luonnontieteissä on perinteisesti pyritti mahdollisimman suureen varmuuteen ja tarkkuuteen niin tietyssä mielessä monimaailmatulkinta (ossa kaikki mahdollinen toteutuu omassa deterministisessä maailmassaan) voi tuntua joidenkin mielestä houkuttelevalta vaihtoehdolta jos tutkijoiden epävarmuuden sietokyky on heikko ja ainoa työkalu on matemaattinen hahmotus determinististen kaavojen avulla."

        MWI:ssä ei voi muodostaa matemaattista alijärjestelmää tai tilannetta, jossa on yksi maailma otettuna useista, ja joka toimii deterministisesti kaavan mukaan. Siksi se on Many-WI eikä One-WI. Työkalujen maininta on väärin siten, että niitä ei ole lisätty, vaan MWI riisuu esim. Kööpenhaminan tulkinnan työkaluista, mutta samalla lisäten jäljellä olevien työkalujen sovelluskertoja. Tai voi sanoa myös entiteettien lukumäärän kasvaneen (missä entiteetti ei ole uudenlainen tai ulkopuolinen; eikä entiteetti ole myöskään maailma).

        B:
        "Monimaailmamalli käytännössä tuottaisi valtavan määrän versumeita jokaisen hiukkastaon vuorovaikutuksen seurauksena ja jo pelkästään logiikan Reductio ad absurdum periaatteen mukaan pitäisi hylätä täysin järjettömänä ja epärealistisena tulkintana."

        Jos esimerkkitodellisuuksia saataisiin ehdottaa vapaasti, niin yhdessä, mikä tehdään MWI-pohjalta, ei ole mitään, mitä ei voisi pitää oikeana todellisuutena. Käytännössä MWI-malli on multiversumi, joka sisältää esim. 16 kompleksilukua eikä mitään muuta ole olemassa. Jos sanotaan, että jonkin tapahtuman jälkeen tässä todellisuudessa on kaksi eri maailmaa hamaan loppuun asti, niin se on silti edelleen esitettävissä 16 kompleksiluvulla. Se ei ole paljon. Lisäksi vastaväittäjä ei pysty koskaan tekemään mallia, jolla on samat ominaisuudet (mutta jota ei sanota MWI:ksi) ja joka voitaisiin esittää alle 16 kompleksiluvulla, joten seurauksena kaikki mallit olisivat väärin jos yksi on sen takia, että sen piti olla 16 tai enemmän.

        2

        Logiikkasi ei ole täysin rakentunut. Sinun pitäisi ehkä selittää millaista se on, että kahden hiukkasen lisäksi on valtava määrä versumeita. Jos teet sen väärin, niin riskinä on, että kun kuvailet kahden hiukkasen väliin versumeita, niin se alkaa kuulosta siltä, mitä MWI ei ole väittänyt. Jos toinen maailma on vain näkymätön ja hajuton entiteetti, jota ei edes pidä ajatella ääneen, niin sillä ei ole edellytyksiä olla myöskään järjenvastainen. Siinä ei ole mitään, mitä vastustaa ja mistä tieto olisi jokaisella jo olemassa (tai tuollaiseen lauseeseen pitää sisältyä itseensä jo jokin 'saat tämän absurditeetin esimerkin nähtäväksesi heti kun teet näin'). Voisit verrata sitä kuitenkin esim. siihen, että väittää olevan olemassa jumalia. Sellainen, mikä ei ole koskaan tehnyt mitään tai ole mitään suhteessa meihin. Kuinka usein logiikka, kuten 'väitteesi on kuin väittäisi olevan olemassa jumalan, joten olet väärässä' toimii normaalille yleisölle?

        B:
        "Einsteinin blokkiuniversumin viitekehyksesä ajalla ei ole merkitystä ja sama koskee kaikkea sellaista ontologiaa jossa ei ole ajan suuntaa, entropiaa eikä selkeää identiteettiä joka alkaa ilmetä vasta kemian ja molekyylien ja makrokappaleiden tasoilla joissa voidaan tunnistaa selkeitä rakenteita."

        Einsteinin blokkiin voidaan laittaa makrokappaleita, ja ne laitetaan mieluiten oikein ja siten, että ne noudattavat luonnonlakeja. Jolloin makrokappaleilla Einsteinin blokissa on varmasti esim. entropian piirteet, heti kun katsoo, paljonko se olisi. Jos esim. identiteetti on vain kappale ja kappalemäärä kysymys, niin ei missään tarvitse määritellä mitään aikaa ollakseen identiteetti jostakin, ja kaikki missä voi sisällyttää kappaleita, on niitä sisältävä.

        Blokissa blokilla on merkitystä. Sillä mitä ajattelet olevan käsitteessä nimeltä 'aika' ei ehkä ole merkitystä blokissa (et varmaan halua kenenkään muun arvioita siitä, kun et kerro). Mutta nyt jos blokki on ehdolla ollakseen kaikki mitä on, niin voi olla, että sillä mitä ajattelet ei ole merkitystä missään.

        "Etenkin, kun meillä on esimerkkejä tilanteista, missä eri historiat vaikuttaisivat reagoivan keskenään kuten aloituksessa mainitussa g-kertoimen teoreettisessa johtamisessa."

        Edelleen väärin. Tässä johdossa ei väitetä olevan romahduksia ja sen jälkneen niiden välttelyä, jotta olisi enemmän kirjoitettavia diagrammeja.

        B:
        "Se epävarmuus kuitenkin alkaa vaikuttaa kun siirrytään sinne kenttien ja hiukkasten ja toisaalta myös elollisen ja elävän luonnon tasolle jotka myös usein hahmotetaan jotenkin satunnaisina vaikka kyse ei ole aidosta satunnaisuudesta..."

        Puhu omasta puolestasi. Sanoisin myös en tuohon liittyen, mutta monien mielestä QM-satunnaisuus on ainoa aito satunnaisuus, koska muut satunnaisuudet ovat epäfyysisiä eivätkä ole olemassa, mikä vie niiden aitoudelta aika paljon.

        B:
        "...vaan siitä että niiden kausaaliketjuja ei voida määritellä fysikaaliselta tasolta lähtien koska ne kykenevät aloittamaan omia kausaaliketjujaan fysikaalisen aineen tasolla mistä seurauksena ihmisen kulttuurin tasolla ovat mm. tekniikan kehitys ja soveltaminen joka on tietysti edelleen kiistanalainen kysymys koska tämä kulttuuri on perinteisesti voimakkaasti sitoutunut deterministiseen fysikalistiseen maailmankuvaan joka on pohjimmiltaan pelkkä ehdollistettu ajatustottumus samalla tavalla kuin uskonnotkin."

        Tieteessä voisi ajatella, että tuo on piilomuuttujan määritelmä, että piilomuuttuja (fyysinen) on subjektiivisesti eri tasolla kuin muuttuja. Ja että jos piilomuuttuja olisi, niin muuttuja ei olisi satunnainen. Kukaan ei silti sanoisi, että se mitä ne tekevät toisilleen, ovat kausaaliketjuja.

        Miksi jotkut historialliset tapahtumat pitää mainita lauseessa, jossa on myös hiukkastasolta alkanut kausaaliketju? Merkitsevätkö nämä tapahtumat jotain enemmän kuin esim. muuttolintujen lento (ovatko ne siis pedagogisesti enemmän opettavia ketjun jäseniä vai ovatko ne jopa teorian kannalta tärkeitä)? Saanko sanoa lauseen, missä on hiukkastason kausaaliketju, ja sellainen tapahtuma kuin 'B:n maailmankuvan muodostuksen alkaminen'.

        B
        "Käytännössä monimaailmatulkinnan suosio johtunee siitä että keskitytään vain siihen omaan universumiin ja unohdetaaan ne vaihtoehtoiset universumit jolloin ei tarvitse opetella mitään uutta ja 1800-luvun lopun deterministinen malli pysyy käytännössä muuttumattomana."

        Onko linkkiä teokseen, missä yhtä maailmaa on millään arviointimenetelmällä enemmän kuin toisia?

        3


      • Anonyymi
        Anonyymi kirjoitti:

        Logiikkasi ei ole täysin rakentunut. Sinun pitäisi ehkä selittää millaista se on, että kahden hiukkasen lisäksi on valtava määrä versumeita. Jos teet sen väärin, niin riskinä on, että kun kuvailet kahden hiukkasen väliin versumeita, niin se alkaa kuulosta siltä, mitä MWI ei ole väittänyt. Jos toinen maailma on vain näkymätön ja hajuton entiteetti, jota ei edes pidä ajatella ääneen, niin sillä ei ole edellytyksiä olla myöskään järjenvastainen. Siinä ei ole mitään, mitä vastustaa ja mistä tieto olisi jokaisella jo olemassa (tai tuollaiseen lauseeseen pitää sisältyä itseensä jo jokin 'saat tämän absurditeetin esimerkin nähtäväksesi heti kun teet näin'). Voisit verrata sitä kuitenkin esim. siihen, että väittää olevan olemassa jumalia. Sellainen, mikä ei ole koskaan tehnyt mitään tai ole mitään suhteessa meihin. Kuinka usein logiikka, kuten 'väitteesi on kuin väittäisi olevan olemassa jumalan, joten olet väärässä' toimii normaalille yleisölle?

        B:
        "Einsteinin blokkiuniversumin viitekehyksesä ajalla ei ole merkitystä ja sama koskee kaikkea sellaista ontologiaa jossa ei ole ajan suuntaa, entropiaa eikä selkeää identiteettiä joka alkaa ilmetä vasta kemian ja molekyylien ja makrokappaleiden tasoilla joissa voidaan tunnistaa selkeitä rakenteita."

        Einsteinin blokkiin voidaan laittaa makrokappaleita, ja ne laitetaan mieluiten oikein ja siten, että ne noudattavat luonnonlakeja. Jolloin makrokappaleilla Einsteinin blokissa on varmasti esim. entropian piirteet, heti kun katsoo, paljonko se olisi. Jos esim. identiteetti on vain kappale ja kappalemäärä kysymys, niin ei missään tarvitse määritellä mitään aikaa ollakseen identiteetti jostakin, ja kaikki missä voi sisällyttää kappaleita, on niitä sisältävä.

        Blokissa blokilla on merkitystä. Sillä mitä ajattelet olevan käsitteessä nimeltä 'aika' ei ehkä ole merkitystä blokissa (et varmaan halua kenenkään muun arvioita siitä, kun et kerro). Mutta nyt jos blokki on ehdolla ollakseen kaikki mitä on, niin voi olla, että sillä mitä ajattelet ei ole merkitystä missään.

        "Etenkin, kun meillä on esimerkkejä tilanteista, missä eri historiat vaikuttaisivat reagoivan keskenään kuten aloituksessa mainitussa g-kertoimen teoreettisessa johtamisessa."

        Edelleen väärin. Tässä johdossa ei väitetä olevan romahduksia ja sen jälkneen niiden välttelyä, jotta olisi enemmän kirjoitettavia diagrammeja.

        B:
        "Se epävarmuus kuitenkin alkaa vaikuttaa kun siirrytään sinne kenttien ja hiukkasten ja toisaalta myös elollisen ja elävän luonnon tasolle jotka myös usein hahmotetaan jotenkin satunnaisina vaikka kyse ei ole aidosta satunnaisuudesta..."

        Puhu omasta puolestasi. Sanoisin myös en tuohon liittyen, mutta monien mielestä QM-satunnaisuus on ainoa aito satunnaisuus, koska muut satunnaisuudet ovat epäfyysisiä eivätkä ole olemassa, mikä vie niiden aitoudelta aika paljon.

        B:
        "...vaan siitä että niiden kausaaliketjuja ei voida määritellä fysikaaliselta tasolta lähtien koska ne kykenevät aloittamaan omia kausaaliketjujaan fysikaalisen aineen tasolla mistä seurauksena ihmisen kulttuurin tasolla ovat mm. tekniikan kehitys ja soveltaminen joka on tietysti edelleen kiistanalainen kysymys koska tämä kulttuuri on perinteisesti voimakkaasti sitoutunut deterministiseen fysikalistiseen maailmankuvaan joka on pohjimmiltaan pelkkä ehdollistettu ajatustottumus samalla tavalla kuin uskonnotkin."

        Tieteessä voisi ajatella, että tuo on piilomuuttujan määritelmä, että piilomuuttuja (fyysinen) on subjektiivisesti eri tasolla kuin muuttuja. Ja että jos piilomuuttuja olisi, niin muuttuja ei olisi satunnainen. Kukaan ei silti sanoisi, että se mitä ne tekevät toisilleen, ovat kausaaliketjuja.

        Miksi jotkut historialliset tapahtumat pitää mainita lauseessa, jossa on myös hiukkastasolta alkanut kausaaliketju? Merkitsevätkö nämä tapahtumat jotain enemmän kuin esim. muuttolintujen lento (ovatko ne siis pedagogisesti enemmän opettavia ketjun jäseniä vai ovatko ne jopa teorian kannalta tärkeitä)? Saanko sanoa lauseen, missä on hiukkastason kausaaliketju, ja sellainen tapahtuma kuin 'B:n maailmankuvan muodostuksen alkaminen'.

        B
        "Käytännössä monimaailmatulkinnan suosio johtunee siitä että keskitytään vain siihen omaan universumiin ja unohdetaaan ne vaihtoehtoiset universumit jolloin ei tarvitse opetella mitään uutta ja 1800-luvun lopun deterministinen malli pysyy käytännössä muuttumattomana."

        Onko linkkiä teokseen, missä yhtä maailmaa on millään arviointimenetelmällä enemmän kuin toisia?

        3

        Kaikissa QM:ään liittyvissä empiirisissä kokeissa, missä on enemmän kuin yksi toisto, on teoriassa katsottu ns. identiteetiltään samanlaista objektia ja sen alkutilaa jatkuvasti uudestaan ja uudestaan. Silloin periaatteessa myös on koettu kaikki maailmat, mitä voi saada aikaiseksi, kun tämä objekti mitataan. Periaatteessa kokeessa ei mainita, mitä siitä tulkitsee, mutta jos väittää, että on ns. suosittu tulkinta, niin tulisi ajatella, mitä se sitten tarkoitaa, että joku sitä tulkintaa mielessään toteuttaa. Ja tässä se on, että koe minkä tein, näytti kaikki maailmat.

        B:
        "Monimaailmatulkinta on tyyppiesimerkki siitä kuinka tiukasti jotkut haluavat takertua joihinkin metafysiikan tason oletuksiin jotka ovat aikoinaan omaksuneet maailmankuvansa perusteiksi eikä se koske pelkästään materialisteja vaan periaatteessa kaikkia jotka ovat samaistuneet johonkin uskomusjärjetelmään josta on tullut olennainen osa heidän omaa identiteettiään eli jos se uskomusjärjestelmä kyseeenalaistetaan niin se koetaan samalla hyökkäyksenä sitä omaa identiteettiä vastaan. Kyse on hyvin yleisestä inhimillisestä ominaisuudesta joka käytännössä voi jumittaa yksilön sekä myös tieteen ja kulttuurin kehitystä."

        Jos jätettäisiin kysymättä ihmisiltä, miksi he tekevät asioita, ja arvioitaisiin heidän tiedostomattomia syitään valita MWI, niin arvaan itse, että syy voi olla se, että MWI:ssä toivotusti kaikki QM:n toimintamuodot palautuvat siihen käsitteistöön, jota QM:ssä on, jolloin malli on täydellinen ja selittää itsensä ja myös käyttönsä. Vähän samalla tavalla voi käydä abiogenesiksen ja jumalan kanssa, että jotain ajateltiin tarvittavan selittämään asioita, mutta sitten yhtäkkiä aine selitti itse biologiaksi muuttumisensa.

        Esimerkki on sinunkin tarkoitukseesi huono, koska niillä jotka eivät valitse MWI:tä on kuitenkin aivan samat metafysiikan pyrkimykset. Eli hekin pyrkivät siihen, että fysiikan asioilla on selitys, joka ei ole mutkia oikova, vaan jossa tuntuu siltä, että asia tiedetään. Ne jotka eivät syleile MWI:tä, eivät ole tästä ajattelusta vapaita, vaan heillä on jokin syy toivoa, että vastaus olisi vielä jokin muu kuin MWI (tai tuossa joutuu ajattelemaan, että vaikka tiedetään, että jotain voi tapahtua MWI-versiona, niin silti ei tiedetä tapahtuuko juuri MWI). Vai oletko sitä mieltä, että ihailet jonkun ei-MWI:n metafyysistä koskemattomuutta? Pystytkö vastamaan kysymykseen, että 'miten paljon materialistisen tiedemiehen metafyysinen puhtaus kasvaa, kun hän vaihtaa näkemyksensä MWI:stä pois shut up and calculateen tms.?' sanomalla, että merkillepantavan paljon'? Jos se ei ole näin, niin silloin tätä mainittua asiaa ei ole hahmotettu tulevaksi ihmiseen MWI:n myötä.

        B:
        "Teor. fysiikassa matemaattinen mielikkuvitus on päästetty valloilleen eli kyse on pohjimmiltaan metafysiikan ja filosofian tason villistä spekulaatiosta joka on piilotettun matemaattisen formalismin alle. Mitää tahansa teoriaa voi yllpitää loputtomiin esim. postuloimalla uusia entieteettejä kuten uusia hiukkasia tai säännönmukaisuuksia jos teoria alkaa näyttää empiirisen tason suhteen epäuskottavalta."

        Jos väität tätä tulevaisuudeksi, niin olet jo olettanut todellisuuden olevan sellainen, että sen voi selittää aina matematiikalla ja hiukkasilla. Tuolle työlle ei siis ole vaihtoehtoa.

        B:
        "Se mikä voi näyttää ns. "lillukanvarsien" tasolla oikealta ja järkevältä voi kokonaisuuden kannalta olla ääliömäisyyden huipentumaa."

        Sellaisesta kokonaistasosta ei ole fysiikassa mitään esimerkkiä, missä ei voisi sanoa vaikuttavan esim. uuden hiukkasen läsnäolo. Emme ole mitenkään valmiita tuomitsemaan alaspäin tältä tasolta lähtien, että miten esim. mikroskooppinen teoria pystyisi muodostamaan sen tapahtumat sen sijasta. Koska sellaista tasoa ei ole annettu, mihin näillä pyrittäisiin.

        4


      • Anonyymi

        ”Ei ole määritelty sellaista ulkopuolista näkökulmaa, joka olisi monien maailmojen ulkopuolella nähden juuri ne kaikki.”

        Kyseessä ei ole mikään mielivaltaisesti määritelty käsite. Ajatelkaamme kvanttimaailman koostuvan hiukkasista, jotka ovat olemassa todennäköisyyspilvinä. Perinteisen tulkinnan mukaan hiukkasen sijainti paikallistuu vasta hiukkasen reagoidessa toisen hiukkasen kanssa. Monimaailma tulkinnan mukaan hiukkanen on olemassa kaikissa sen mahdollisissa (potentiaalisissa) sijainneissa ja sen sijainti ei paikallistu reaktiossa, vaan kaikki mahdolliset historiat eli reaktioketjut elävät rinnakkain. Tämä muodostaa lukuisten mahdollisten historioiden verkoston ja aikaa mitataan tuon järjestelmän reaktioiden suhteella.

        B-piti yllä olevaa näkemystä monimutkaisena, mutta loppujen lopuksi se on kaikkein luonnollisin, koska siinä ei tapahdu ”arpomista”, missä joistakin mahdollisuuksista pitäisi jotenkin mystisesti valita yksi ja ainoa. Einstein oli tuota ”arpomista” vastaan ja totesi: ”Jumala ei heitä noppaa universumilla.” Mutta tietääkseni Einsteinin aikoina monimaailma-tulkinta ei ollut esillä.

        Monet ”arpomista” vastustaneet fyysikot kallistuivat enemmän piilomuuttujien puolelle eli ajatukseen, että arpominen on vain näennäinen illuusio, mikä johtuu siitä, ettemme tiedä järjestelmän kaikkia sisäisiä parametreja tarkasti. Tämä piilomuuttuja-ajatus ei ymmärtääkseni ole nykyisin fyysikkojen suosiossa, koska sen kautta ei pystytä selittämään kvanttimekaniikan erikoisuuksia.

        Kuuluisan kaksoisrakokokeen tulos selitetään monimaailmatulkinnan kautta siten, että historiat voivat reagoida keskenään - ja ne erkanevat peruuttamattomasti vasta, jos niiden yhdistyminen 100% identtiseksi tulevaisuuden tilaksi on mahdotonta.

        Ulkopuolinen näkökulma on sellainen, millä ei ole minkäänlaista kausaalista yhteyttä tuon useita historioita sisältävän järjestelmän kanssa. Käytännössä tuo on aika vaikeaa, koska mikään systeemi ei ole täysin eristetty ympäristöstään muuten kuin hyvin lyhyitä aikoja.


      • Anonyymi

        ”Ei ole määritelty sellaista ulkopuolista näkökulmaa, joka olisi monien maailmojen ulkopuolella nähden juuri ne kaikki.”

        Kyllä on määritelty.

        Ulkopuolinen näkökulma on sellainen, joka ei ole missään kausaalisessa yhteydessä (edes hiukkastason reaktioissa) itse systeemin kanssa.

        Käytännössä makrotason systeemiä on mahdoton eristää täysin, joten asiaa voidaan tutkia vain mikrotasolla ja sielläkin systeemin eristäminen pitkäksi aikaa lienee mahdotonta.

        Käytännön koejärjestely ja tulokset on kuvattu:
        https://arxiv.org/abs/1310.4691

        Tässä ilmiö väitetysti todennettiin käyttäen fotoneja.

        Minä taistelen vielä yrittäessäni ymmärtää tuon paperin keskeistä sisältöä. Yhteenveto on selkeä, mutta koejärjestely ja siitä vedetty johtopäätös ei.


      • Anonyymi
        Anonyymi kirjoitti:

        Logiikkasi ei ole täysin rakentunut. Sinun pitäisi ehkä selittää millaista se on, että kahden hiukkasen lisäksi on valtava määrä versumeita. Jos teet sen väärin, niin riskinä on, että kun kuvailet kahden hiukkasen väliin versumeita, niin se alkaa kuulosta siltä, mitä MWI ei ole väittänyt. Jos toinen maailma on vain näkymätön ja hajuton entiteetti, jota ei edes pidä ajatella ääneen, niin sillä ei ole edellytyksiä olla myöskään järjenvastainen. Siinä ei ole mitään, mitä vastustaa ja mistä tieto olisi jokaisella jo olemassa (tai tuollaiseen lauseeseen pitää sisältyä itseensä jo jokin 'saat tämän absurditeetin esimerkin nähtäväksesi heti kun teet näin'). Voisit verrata sitä kuitenkin esim. siihen, että väittää olevan olemassa jumalia. Sellainen, mikä ei ole koskaan tehnyt mitään tai ole mitään suhteessa meihin. Kuinka usein logiikka, kuten 'väitteesi on kuin väittäisi olevan olemassa jumalan, joten olet väärässä' toimii normaalille yleisölle?

        B:
        "Einsteinin blokkiuniversumin viitekehyksesä ajalla ei ole merkitystä ja sama koskee kaikkea sellaista ontologiaa jossa ei ole ajan suuntaa, entropiaa eikä selkeää identiteettiä joka alkaa ilmetä vasta kemian ja molekyylien ja makrokappaleiden tasoilla joissa voidaan tunnistaa selkeitä rakenteita."

        Einsteinin blokkiin voidaan laittaa makrokappaleita, ja ne laitetaan mieluiten oikein ja siten, että ne noudattavat luonnonlakeja. Jolloin makrokappaleilla Einsteinin blokissa on varmasti esim. entropian piirteet, heti kun katsoo, paljonko se olisi. Jos esim. identiteetti on vain kappale ja kappalemäärä kysymys, niin ei missään tarvitse määritellä mitään aikaa ollakseen identiteetti jostakin, ja kaikki missä voi sisällyttää kappaleita, on niitä sisältävä.

        Blokissa blokilla on merkitystä. Sillä mitä ajattelet olevan käsitteessä nimeltä 'aika' ei ehkä ole merkitystä blokissa (et varmaan halua kenenkään muun arvioita siitä, kun et kerro). Mutta nyt jos blokki on ehdolla ollakseen kaikki mitä on, niin voi olla, että sillä mitä ajattelet ei ole merkitystä missään.

        "Etenkin, kun meillä on esimerkkejä tilanteista, missä eri historiat vaikuttaisivat reagoivan keskenään kuten aloituksessa mainitussa g-kertoimen teoreettisessa johtamisessa."

        Edelleen väärin. Tässä johdossa ei väitetä olevan romahduksia ja sen jälkneen niiden välttelyä, jotta olisi enemmän kirjoitettavia diagrammeja.

        B:
        "Se epävarmuus kuitenkin alkaa vaikuttaa kun siirrytään sinne kenttien ja hiukkasten ja toisaalta myös elollisen ja elävän luonnon tasolle jotka myös usein hahmotetaan jotenkin satunnaisina vaikka kyse ei ole aidosta satunnaisuudesta..."

        Puhu omasta puolestasi. Sanoisin myös en tuohon liittyen, mutta monien mielestä QM-satunnaisuus on ainoa aito satunnaisuus, koska muut satunnaisuudet ovat epäfyysisiä eivätkä ole olemassa, mikä vie niiden aitoudelta aika paljon.

        B:
        "...vaan siitä että niiden kausaaliketjuja ei voida määritellä fysikaaliselta tasolta lähtien koska ne kykenevät aloittamaan omia kausaaliketjujaan fysikaalisen aineen tasolla mistä seurauksena ihmisen kulttuurin tasolla ovat mm. tekniikan kehitys ja soveltaminen joka on tietysti edelleen kiistanalainen kysymys koska tämä kulttuuri on perinteisesti voimakkaasti sitoutunut deterministiseen fysikalistiseen maailmankuvaan joka on pohjimmiltaan pelkkä ehdollistettu ajatustottumus samalla tavalla kuin uskonnotkin."

        Tieteessä voisi ajatella, että tuo on piilomuuttujan määritelmä, että piilomuuttuja (fyysinen) on subjektiivisesti eri tasolla kuin muuttuja. Ja että jos piilomuuttuja olisi, niin muuttuja ei olisi satunnainen. Kukaan ei silti sanoisi, että se mitä ne tekevät toisilleen, ovat kausaaliketjuja.

        Miksi jotkut historialliset tapahtumat pitää mainita lauseessa, jossa on myös hiukkastasolta alkanut kausaaliketju? Merkitsevätkö nämä tapahtumat jotain enemmän kuin esim. muuttolintujen lento (ovatko ne siis pedagogisesti enemmän opettavia ketjun jäseniä vai ovatko ne jopa teorian kannalta tärkeitä)? Saanko sanoa lauseen, missä on hiukkastason kausaaliketju, ja sellainen tapahtuma kuin 'B:n maailmankuvan muodostuksen alkaminen'.

        B
        "Käytännössä monimaailmatulkinnan suosio johtunee siitä että keskitytään vain siihen omaan universumiin ja unohdetaaan ne vaihtoehtoiset universumit jolloin ei tarvitse opetella mitään uutta ja 1800-luvun lopun deterministinen malli pysyy käytännössä muuttumattomana."

        Onko linkkiä teokseen, missä yhtä maailmaa on millään arviointimenetelmällä enemmän kuin toisia?

        3

        ”””
        "Etenkin, kun meillä on esimerkkejä tilanteista, missä eri historiat vaikuttaisivat reagoivan keskenään kuten aloituksessa mainitussa g-kertoimen teoreettisessa johtamisessa."

        Edelleen väärin. Tässä johdossa ei väitetä olevan romahduksia ja sen jälkneen niiden välttelyä, jotta olisi enemmän kirjoitettavia diagrammeja.
        ”””
        Minä puolustin monimaailmatulkintaa, ja siinä ei ole romahduksia (kuten totesit). Mikä tässä siis oli väärin?

        Tämä oli esimerkki siitä, miten useat rinnakkaiset historiat (Feynmanin diagrammin loputtomat mahdolliset ketjut) yhdistyivät.

        Kööpenhaminan tulkinnassa historiat eivät voi reagoida keskenään, koska lukuisia historioita ei synny aaltofunktiont romahtaessa. Monimaailmassa taas eri historiat elävät rinnakkain ja voivat reagoida keskenään, jos historiat johtavat identtiseen lopputulokseen kuten g-kertoimen johto-esimerkissä.

        Ulkopuolelta katsoen mikä tahansa historioista on mahdollinen ja meillä ei vaikuttaisi olevan mahdollista erottaa, mikä niistä toteutui. (Jokaisen alkutila ja lopputila on identtinen.) Yllättävää on, että tässä johdossa vaikuttaa, että kaikki mahdolliset historiat toteutuivat samanaikaisesti - ja näemme kaikkien mahdollisten historioiden summan. Toki kyse on virtuaalihiukkasista, mutta minulla herää kysymys antaako tämä jotain syvempää tietoa, miten maailma toimii.


      • Anonyymi
        Anonyymi kirjoitti:

        ”Ei ole määritelty sellaista ulkopuolista näkökulmaa, joka olisi monien maailmojen ulkopuolella nähden juuri ne kaikki.”

        Kyllä on määritelty.

        Ulkopuolinen näkökulma on sellainen, joka ei ole missään kausaalisessa yhteydessä (edes hiukkastason reaktioissa) itse systeemin kanssa.

        Käytännössä makrotason systeemiä on mahdoton eristää täysin, joten asiaa voidaan tutkia vain mikrotasolla ja sielläkin systeemin eristäminen pitkäksi aikaa lienee mahdotonta.

        Käytännön koejärjestely ja tulokset on kuvattu:
        https://arxiv.org/abs/1310.4691

        Tässä ilmiö väitetysti todennettiin käyttäen fotoneja.

        Minä taistelen vielä yrittäessäni ymmärtää tuon paperin keskeistä sisältöä. Yhteenveto on selkeä, mutta koejärjestely ja siitä vedetty johtopäätös ei.

        "Kyseessä ei ole mikään mielivaltaisesti määritelty käsite. ... Tämä muodostaa lukuisten mahdollisten historioiden verkoston ja aikaa mitataan tuon järjestelmän reaktioiden suhteella."

        Olet määritellyt olevasi sisäinen näkökulma.

        "Kuuluisan kaksoisrakokokeen tulos selitetään monimaailmatulkinnan kautta siten, että historiat voivat reagoida keskenään - ja ne erkanevat peruuttamattomasti vasta, jos niiden yhdistyminen 100% identtiseksi tulevaisuuden tilaksi on mahdotonta."

        Jos haluaa kaksoisrakokokeen tuottavan pisteitä kuvalevylle, niin silloin on välttämätöntä sanoa, että maailmaksi nimitetyt historiat eivät vaikuta toisiinsa enää mitentkään, vaan että piste jää sinne minne se ensin syntyi.

        Tullakseen pisteeksi, pisteen ei tarvitse olla vielä olemassa. Kun sanotaan piste voidaan ajatella, että se on muiden asioiden lisäksi yhden saapuvan hiukkasen aaltofunktion yksi kohta ja sijainti avaruudessa. Kun MWI:ssä sanotaan maailma ja että on korrelaatiota havaitsijan tilan ja saapuvan hiukkasen tilojen välillä, niin tämä piste avaruudessa on se Hilbertin avaruuden suunta, johon havaitsija korreloituu (hänestä näyttää Kööpenhaminalaisesti kuin piste olisi täytetty todennäköisyydellä 1), kun hän sitä myöhemmin pitää tuloksena. Nyt voidaan sanoa, että ei pidä paikkansa että tämä avaruuden piste ja siinä oleva aaltofunktio eri hetkinä ja alkaen alussa todennäköisyydellä 0 olisi reagoinut minkään kanssa. Paitsi aalto tässä pisteessä on reagoinut jotenkin mainitsemattomalla tavalla sellaisen materian kanssa, mitä levyssä on. Väitit kuitenkin, että aaltofunktio reagoisi ajan ja avaruuden yli aaltoon, jonka varsinainen aaltomassa on vasta tulossa kohti levyä, tai että levyllä olevilla pisteillä, jotka seuraavaksi ovat selvästi eri maailmoja, olisi tekemistä keskenään kun niissä on osa aallosta luonaan.

        Ennen saapumista levylle ja mittaukseen ei myöskään tulisi puhua mainittavalla tavalla mistään reagoinnista. Kun kaksoisrakokoe esitetään kahtena palloaaltona, siinä puhutaan tiloista, jotka ovat ikään kuin jaettuna kahtia. Jotkut pitävät näiden tilojen historioita erillisinä ja puhuvat niiden sekoittamisesta. Levyllä meidän tulee kuitenkin jakaa ongelma tiloihin, jotka ovat yksittäisiä pisteitä. Näilläkin pisteillä on historia joka ajan hetkellä. Kun kahdesta pallosta tai mistä hyvänsä tulee funktion amplitudia näiden pisteiden luokse, kyseessä ei ole minkään reagoiminen mihinkään, vaan kyseessä on sellainen odottelu, missä jonkin funktion odotetaan kehittyvän ajassa sellaiseksi, että näiden pisteiden luona funktio on jotain muuta kuin nolla, mikä ei koske niitä kaikkia. Minkä tahansa QM-tulkinnan esitys siitä, mitä mittauksessa tapahtuu, on vaillinainen, eikä sille ole esim. selitystä, joka etenisi tällaisena funktiona ajassa, vaikka MWI sitä haluaisi olevansa. Olisi myös huomattavasti kätevämpää puhua asioista ilman kahta rakoa aaltojen tiellä.

        Äsken olleet palloaallot voivat merkitä jonkun mielestä maailmoja myös, mutta silloin maailmat ovat vain miniskulaarista osatietoa hiukkasen menneistä ajoista (vaikka ovatkin tiloja ja tilat ovat aina yhtä täysiä tiloja kuin muutkin). Lisäksi kukaan havaitsija ei yleensä mittaa kumpaakan näistä maailmoista omakseen. Ja tällaisessa maailmassa maailma ei jakaudu niihin vaan on jäänyt jakautumatta. Normaalissa kokeessa ei pitäisi pitää paikkaansa, että palloaallot olisivat maailmat, jotka olisivat syntyneet hetkeksi ja tuotu yhteen. Missä rako olisi se, missä jako olisi tapahtunut, mutta joka olisi myös tajunnut ettei jaetakaan.

        "Ulkopuolinen näkökulma on sellainen, millä ei ole minkäänlaista kausaalista yhteyttä tuon useita historioita sisältävän järjestelmän kanssa."

        Kyseessä oli linkeissä kietoutunut yhteys ja sen puuttuminen. Kietoutunut yhteys ei ole kausaalinen.

        "Kyllä on määritelty.
        Ulkopuolinen näkökulma on sellainen, joka ei ole missään kausaalisessa yhteydessä (edes hiukkastason reaktioissa) itse systeemin kanssa."

        Nyt kun olet ulkopuolella, etkä ilmeisesti näe aikaa, niin määrittele maailmoita.

        "Käytännön koejärjestely ja tulokset on kuvattu:"

        Artikkeli ei määritellyt useita maailmoita. Mutta käytti suhteellisia tiloja ollessaan systeemin sisällä.

        Tässä on kirjoitettu pidempi review teorialle.
        https://arxiv.org/pdf/1610.04773.pdf


      • Anonyymi
        Anonyymi kirjoitti:

        "Kyseessä ei ole mikään mielivaltaisesti määritelty käsite. ... Tämä muodostaa lukuisten mahdollisten historioiden verkoston ja aikaa mitataan tuon järjestelmän reaktioiden suhteella."

        Olet määritellyt olevasi sisäinen näkökulma.

        "Kuuluisan kaksoisrakokokeen tulos selitetään monimaailmatulkinnan kautta siten, että historiat voivat reagoida keskenään - ja ne erkanevat peruuttamattomasti vasta, jos niiden yhdistyminen 100% identtiseksi tulevaisuuden tilaksi on mahdotonta."

        Jos haluaa kaksoisrakokokeen tuottavan pisteitä kuvalevylle, niin silloin on välttämätöntä sanoa, että maailmaksi nimitetyt historiat eivät vaikuta toisiinsa enää mitentkään, vaan että piste jää sinne minne se ensin syntyi.

        Tullakseen pisteeksi, pisteen ei tarvitse olla vielä olemassa. Kun sanotaan piste voidaan ajatella, että se on muiden asioiden lisäksi yhden saapuvan hiukkasen aaltofunktion yksi kohta ja sijainti avaruudessa. Kun MWI:ssä sanotaan maailma ja että on korrelaatiota havaitsijan tilan ja saapuvan hiukkasen tilojen välillä, niin tämä piste avaruudessa on se Hilbertin avaruuden suunta, johon havaitsija korreloituu (hänestä näyttää Kööpenhaminalaisesti kuin piste olisi täytetty todennäköisyydellä 1), kun hän sitä myöhemmin pitää tuloksena. Nyt voidaan sanoa, että ei pidä paikkansa että tämä avaruuden piste ja siinä oleva aaltofunktio eri hetkinä ja alkaen alussa todennäköisyydellä 0 olisi reagoinut minkään kanssa. Paitsi aalto tässä pisteessä on reagoinut jotenkin mainitsemattomalla tavalla sellaisen materian kanssa, mitä levyssä on. Väitit kuitenkin, että aaltofunktio reagoisi ajan ja avaruuden yli aaltoon, jonka varsinainen aaltomassa on vasta tulossa kohti levyä, tai että levyllä olevilla pisteillä, jotka seuraavaksi ovat selvästi eri maailmoja, olisi tekemistä keskenään kun niissä on osa aallosta luonaan.

        Ennen saapumista levylle ja mittaukseen ei myöskään tulisi puhua mainittavalla tavalla mistään reagoinnista. Kun kaksoisrakokoe esitetään kahtena palloaaltona, siinä puhutaan tiloista, jotka ovat ikään kuin jaettuna kahtia. Jotkut pitävät näiden tilojen historioita erillisinä ja puhuvat niiden sekoittamisesta. Levyllä meidän tulee kuitenkin jakaa ongelma tiloihin, jotka ovat yksittäisiä pisteitä. Näilläkin pisteillä on historia joka ajan hetkellä. Kun kahdesta pallosta tai mistä hyvänsä tulee funktion amplitudia näiden pisteiden luokse, kyseessä ei ole minkään reagoiminen mihinkään, vaan kyseessä on sellainen odottelu, missä jonkin funktion odotetaan kehittyvän ajassa sellaiseksi, että näiden pisteiden luona funktio on jotain muuta kuin nolla, mikä ei koske niitä kaikkia. Minkä tahansa QM-tulkinnan esitys siitä, mitä mittauksessa tapahtuu, on vaillinainen, eikä sille ole esim. selitystä, joka etenisi tällaisena funktiona ajassa, vaikka MWI sitä haluaisi olevansa. Olisi myös huomattavasti kätevämpää puhua asioista ilman kahta rakoa aaltojen tiellä.

        Äsken olleet palloaallot voivat merkitä jonkun mielestä maailmoja myös, mutta silloin maailmat ovat vain miniskulaarista osatietoa hiukkasen menneistä ajoista (vaikka ovatkin tiloja ja tilat ovat aina yhtä täysiä tiloja kuin muutkin). Lisäksi kukaan havaitsija ei yleensä mittaa kumpaakan näistä maailmoista omakseen. Ja tällaisessa maailmassa maailma ei jakaudu niihin vaan on jäänyt jakautumatta. Normaalissa kokeessa ei pitäisi pitää paikkaansa, että palloaallot olisivat maailmat, jotka olisivat syntyneet hetkeksi ja tuotu yhteen. Missä rako olisi se, missä jako olisi tapahtunut, mutta joka olisi myös tajunnut ettei jaetakaan.

        "Ulkopuolinen näkökulma on sellainen, millä ei ole minkäänlaista kausaalista yhteyttä tuon useita historioita sisältävän järjestelmän kanssa."

        Kyseessä oli linkeissä kietoutunut yhteys ja sen puuttuminen. Kietoutunut yhteys ei ole kausaalinen.

        "Kyllä on määritelty.
        Ulkopuolinen näkökulma on sellainen, joka ei ole missään kausaalisessa yhteydessä (edes hiukkastason reaktioissa) itse systeemin kanssa."

        Nyt kun olet ulkopuolella, etkä ilmeisesti näe aikaa, niin määrittele maailmoita.

        "Käytännön koejärjestely ja tulokset on kuvattu:"

        Artikkeli ei määritellyt useita maailmoita. Mutta käytti suhteellisia tiloja ollessaan systeemin sisällä.

        Tässä on kirjoitettu pidempi review teorialle.
        https://arxiv.org/pdf/1610.04773.pdf

        "Minä puolustin monimaailmatulkintaa, ja siinä ei ole romahduksia (kuten totesit). Mikä tässä siis oli väärin?"

        Jos se pitää kirjoittaa MWI:ksi, niin mikään ei ole korreloitu havaitsijaan tai ulkopuoliseen objektiin.

        "Tämä oli esimerkki siitä, miten useat rinnakkaiset historiat (Feynmanin diagrammin loputtomat mahdolliset ketjut) yhdistyivät."

        Niissä voisi Fock-avaruudelle väittää olevan viimeisen hetken yhdistymä, kun kaikkien Fock-avaruuksien loppu on sama. Kun Fock-avaruuden alkukin on sama, niin tästä ei seuraa yhtä perusteltua väitettä sille, että olisi ollut kyseessä tila, josta tuli joskus eri tila kuin muista, ennenkuin pääsi lopputilaan. Lisäksi jos tarkastellaan suunta- ja nopeusriippuvaa vaikutusalaa, niin se voi muuttua diagrammien vaihtamisen myötä merkiten juuri jotain muuta kuin 'diagrammeista huolimatta yhdistämistä'.

        Feynmanin diagrammit eivät ole minkään tilan esityksiä tai tilan jakamista sen tilan eri osiin, joista tulisi esim. superpositio tai monta historiaa dekoheroituneena. Diagrammit muodostuvat sarjakehitelmästä, joka on tehty operaattorin-kaltaiselle objektille, joka operaattori toimii tapauksessa olevien oikeiden tilojen välillä silloin, kun se muuttuu. Tosin tila ja operaattorit eivät ole oikeaa QFT-kieltä enää silloin.

        "Monimaailmassa taas eri historiat elävät rinnakkain ja voivat reagoida keskenään, jos historiat johtavat identtiseen lopputulokseen kuten g-kertoimen johto-esimerkissä."

        Kun jokin reagoi jonkun kanssa ,sillä on jokin tulos, eikä reagointi ole päällä perustuen lopputuloksiin (paitsi silloin, kun puhutaan säilyvistä suureista tai jostain violaatiosta, millaisia MWI ei ole esittänyt).

        Jos ajattelet aaltojen interferenssikuviota (3D:nä ja keskellä tyhjyyttä), niin onko siinä 'sama lopputulos' se, että aallot kulkevat saman pisteen kautta? MItä tapahtuu kaikkialla koko ajan? Tottakai se on määritelmä, että aallot lasketaan yhteen yhdessä pisteessä ja summa on interferenssi. Tällä ei ole mitään vaikutusta sille, mitä MWI:ssä tulisi tehdä, eikä varmastikaan senkään kanssa, miten pienoiskoossakaan maailmoita peruutettaisiin useasta maailmasta yhteen. Siinä siis riittää pelkkä korrelaation poisto, ja lopputulokset joihin osoitetaan voivat olla kaikki aivan levällään, jolloin havaitsija luulee, että jokin on muuttunut superpositioksi.


      • Anonyymi
        Anonyymi kirjoitti:

        "Kyseessä ei ole mikään mielivaltaisesti määritelty käsite. ... Tämä muodostaa lukuisten mahdollisten historioiden verkoston ja aikaa mitataan tuon järjestelmän reaktioiden suhteella."

        Olet määritellyt olevasi sisäinen näkökulma.

        "Kuuluisan kaksoisrakokokeen tulos selitetään monimaailmatulkinnan kautta siten, että historiat voivat reagoida keskenään - ja ne erkanevat peruuttamattomasti vasta, jos niiden yhdistyminen 100% identtiseksi tulevaisuuden tilaksi on mahdotonta."

        Jos haluaa kaksoisrakokokeen tuottavan pisteitä kuvalevylle, niin silloin on välttämätöntä sanoa, että maailmaksi nimitetyt historiat eivät vaikuta toisiinsa enää mitentkään, vaan että piste jää sinne minne se ensin syntyi.

        Tullakseen pisteeksi, pisteen ei tarvitse olla vielä olemassa. Kun sanotaan piste voidaan ajatella, että se on muiden asioiden lisäksi yhden saapuvan hiukkasen aaltofunktion yksi kohta ja sijainti avaruudessa. Kun MWI:ssä sanotaan maailma ja että on korrelaatiota havaitsijan tilan ja saapuvan hiukkasen tilojen välillä, niin tämä piste avaruudessa on se Hilbertin avaruuden suunta, johon havaitsija korreloituu (hänestä näyttää Kööpenhaminalaisesti kuin piste olisi täytetty todennäköisyydellä 1), kun hän sitä myöhemmin pitää tuloksena. Nyt voidaan sanoa, että ei pidä paikkansa että tämä avaruuden piste ja siinä oleva aaltofunktio eri hetkinä ja alkaen alussa todennäköisyydellä 0 olisi reagoinut minkään kanssa. Paitsi aalto tässä pisteessä on reagoinut jotenkin mainitsemattomalla tavalla sellaisen materian kanssa, mitä levyssä on. Väitit kuitenkin, että aaltofunktio reagoisi ajan ja avaruuden yli aaltoon, jonka varsinainen aaltomassa on vasta tulossa kohti levyä, tai että levyllä olevilla pisteillä, jotka seuraavaksi ovat selvästi eri maailmoja, olisi tekemistä keskenään kun niissä on osa aallosta luonaan.

        Ennen saapumista levylle ja mittaukseen ei myöskään tulisi puhua mainittavalla tavalla mistään reagoinnista. Kun kaksoisrakokoe esitetään kahtena palloaaltona, siinä puhutaan tiloista, jotka ovat ikään kuin jaettuna kahtia. Jotkut pitävät näiden tilojen historioita erillisinä ja puhuvat niiden sekoittamisesta. Levyllä meidän tulee kuitenkin jakaa ongelma tiloihin, jotka ovat yksittäisiä pisteitä. Näilläkin pisteillä on historia joka ajan hetkellä. Kun kahdesta pallosta tai mistä hyvänsä tulee funktion amplitudia näiden pisteiden luokse, kyseessä ei ole minkään reagoiminen mihinkään, vaan kyseessä on sellainen odottelu, missä jonkin funktion odotetaan kehittyvän ajassa sellaiseksi, että näiden pisteiden luona funktio on jotain muuta kuin nolla, mikä ei koske niitä kaikkia. Minkä tahansa QM-tulkinnan esitys siitä, mitä mittauksessa tapahtuu, on vaillinainen, eikä sille ole esim. selitystä, joka etenisi tällaisena funktiona ajassa, vaikka MWI sitä haluaisi olevansa. Olisi myös huomattavasti kätevämpää puhua asioista ilman kahta rakoa aaltojen tiellä.

        Äsken olleet palloaallot voivat merkitä jonkun mielestä maailmoja myös, mutta silloin maailmat ovat vain miniskulaarista osatietoa hiukkasen menneistä ajoista (vaikka ovatkin tiloja ja tilat ovat aina yhtä täysiä tiloja kuin muutkin). Lisäksi kukaan havaitsija ei yleensä mittaa kumpaakan näistä maailmoista omakseen. Ja tällaisessa maailmassa maailma ei jakaudu niihin vaan on jäänyt jakautumatta. Normaalissa kokeessa ei pitäisi pitää paikkaansa, että palloaallot olisivat maailmat, jotka olisivat syntyneet hetkeksi ja tuotu yhteen. Missä rako olisi se, missä jako olisi tapahtunut, mutta joka olisi myös tajunnut ettei jaetakaan.

        "Ulkopuolinen näkökulma on sellainen, millä ei ole minkäänlaista kausaalista yhteyttä tuon useita historioita sisältävän järjestelmän kanssa."

        Kyseessä oli linkeissä kietoutunut yhteys ja sen puuttuminen. Kietoutunut yhteys ei ole kausaalinen.

        "Kyllä on määritelty.
        Ulkopuolinen näkökulma on sellainen, joka ei ole missään kausaalisessa yhteydessä (edes hiukkastason reaktioissa) itse systeemin kanssa."

        Nyt kun olet ulkopuolella, etkä ilmeisesti näe aikaa, niin määrittele maailmoita.

        "Käytännön koejärjestely ja tulokset on kuvattu:"

        Artikkeli ei määritellyt useita maailmoita. Mutta käytti suhteellisia tiloja ollessaan systeemin sisällä.

        Tässä on kirjoitettu pidempi review teorialle.
        https://arxiv.org/pdf/1610.04773.pdf

        Kiitos linkistä - täytyy lukea paremmalla ajalla. Pyrimme ehkä kuvaamaan liikaa verbaalisesti. Minulla on ainakin vaikeuksia seurata ajatuksenkulkuasi… Pitää varmaan tavata teksti uudelleen lause kerrallaan.

        ”Olet määritellyt olevasi sisäinen näkökulma.”

        Kokeen lopputulos riippuu siitä oletko sen kanssa yhteydessä vai ei. Tämä ei ole pelkkä määritelmä, vaan se vaikuttaa esimerkiksi kaksoisrakokokeessa loppuhavaintoon.

        MWI kaksoisrako-kokeen tulkinta on nähdäkseni seuraava:

        Jos mittaat kumman raon läpi elektroni (tms) kulki, niin interferenssi kuviota ei synny. Olet ollut kokeen kanssa kausaalisessa yhteydessä, jolloin havainnoit vain yhden historian - eivätkä mahdolliset potentiaaliset historiat reagoi keskenään. Jos taas et mittaa, minkä raon läpi elektronit menevät, olet systeemin ulkopuolella ja havaitset interferenssi-kuvion - eli kaikkien mahdollisten elektronin rata-historioiden yhteisvaikutuksen.

        Taustalevyssä jokainen elektroni realisoituu tietenkin vain yhteen pisteeseen, mutta tässä vaiheessa olet jälleen kausaalisessa yhteydessä mittaukseen: Tarkkailet taustalevyä. Mutta kun et sekoittanut itseäsi itse elektronin lentoradan historiaan havainnoit taustalevyllä interferenssikuvion, mikä syntyy mahdollisten historioiden yhteisvaikutuksesta.

        (Tämä on vastaava ilmiö kuin edellä mainitun g-kertoimen johdossa Feynmanin diagrammesta. Näemme kaikkien mahdollisten historioiden yhteisvaikutuksen.)

        MWI:ssä todennäköisyysfunktio kuvaa mahdollisten historioiden nippua. Todennäköisfunktion aallot ovat eri potentiaalisten historioiden todennäköisyystiheyksiä, ja koska nämä aallot eli hiukkasen mahdolliset radat (historiat) kykenevät reagoimaan keskenään, voimme tulkita, että nämä kaikki historiat ovat todellisia.

        Monimaailmatulkinnasta seuraa myös ennustuksia. Jos pystyisit eristämään itsesi taustalevystä täysin, niin ettei elektronin siihen edellä aktualisoitunut sijainti vaikuttaisi systeemiisi (sinuun) mitenkään (edes epäsuorasti), niin periaatteessa aktualisoitumista ei tapahtuisi ja kaikkien mahdollisten historioiden ketju eläisi sinun näkökulmasta vieläkin pidemmälle.


      • Anonyymi
        Anonyymi kirjoitti:

        "Minä puolustin monimaailmatulkintaa, ja siinä ei ole romahduksia (kuten totesit). Mikä tässä siis oli väärin?"

        Jos se pitää kirjoittaa MWI:ksi, niin mikään ei ole korreloitu havaitsijaan tai ulkopuoliseen objektiin.

        "Tämä oli esimerkki siitä, miten useat rinnakkaiset historiat (Feynmanin diagrammin loputtomat mahdolliset ketjut) yhdistyivät."

        Niissä voisi Fock-avaruudelle väittää olevan viimeisen hetken yhdistymä, kun kaikkien Fock-avaruuksien loppu on sama. Kun Fock-avaruuden alkukin on sama, niin tästä ei seuraa yhtä perusteltua väitettä sille, että olisi ollut kyseessä tila, josta tuli joskus eri tila kuin muista, ennenkuin pääsi lopputilaan. Lisäksi jos tarkastellaan suunta- ja nopeusriippuvaa vaikutusalaa, niin se voi muuttua diagrammien vaihtamisen myötä merkiten juuri jotain muuta kuin 'diagrammeista huolimatta yhdistämistä'.

        Feynmanin diagrammit eivät ole minkään tilan esityksiä tai tilan jakamista sen tilan eri osiin, joista tulisi esim. superpositio tai monta historiaa dekoheroituneena. Diagrammit muodostuvat sarjakehitelmästä, joka on tehty operaattorin-kaltaiselle objektille, joka operaattori toimii tapauksessa olevien oikeiden tilojen välillä silloin, kun se muuttuu. Tosin tila ja operaattorit eivät ole oikeaa QFT-kieltä enää silloin.

        "Monimaailmassa taas eri historiat elävät rinnakkain ja voivat reagoida keskenään, jos historiat johtavat identtiseen lopputulokseen kuten g-kertoimen johto-esimerkissä."

        Kun jokin reagoi jonkun kanssa ,sillä on jokin tulos, eikä reagointi ole päällä perustuen lopputuloksiin (paitsi silloin, kun puhutaan säilyvistä suureista tai jostain violaatiosta, millaisia MWI ei ole esittänyt).

        Jos ajattelet aaltojen interferenssikuviota (3D:nä ja keskellä tyhjyyttä), niin onko siinä 'sama lopputulos' se, että aallot kulkevat saman pisteen kautta? MItä tapahtuu kaikkialla koko ajan? Tottakai se on määritelmä, että aallot lasketaan yhteen yhdessä pisteessä ja summa on interferenssi. Tällä ei ole mitään vaikutusta sille, mitä MWI:ssä tulisi tehdä, eikä varmastikaan senkään kanssa, miten pienoiskoossakaan maailmoita peruutettaisiin useasta maailmasta yhteen. Siinä siis riittää pelkkä korrelaation poisto, ja lopputulokset joihin osoitetaan voivat olla kaikki aivan levällään, jolloin havaitsija luulee, että jokin on muuttunut superpositioksi.

        ”Kun jokin reagoi jonkun kanssa ,sillä on jokin tulos, eikä reagointi ole päällä perustuen lopputuloksiin (paitsi silloin, kun puhutaan säilyvistä suureista tai jostain violaatiosta, millaisia MWI ei ole esittänyt).”

        Kvanttimekaniikassa MWI-tulkinnassa ei ole mitään tulosta, vaan kaikki mahdolliset historiat ovat olemassa todennäköisyysfunktiona. Tulos aktualisoituu havainnoitsijalle vasta, kun olet kausaalisessa yhteydessä systeemiin, jonka jälkeen sinä olet väistämättä osa yhtä historiaa. Tämä ei tarkoita, etteivätkö muutkin mahdolliset historiat, missä sinä ole, jatkaisi olemassa oloaan ihan yhtä lailla. Se historia, joka on aktualisoitunut sinulle, on vielä määrittelemätön niille, joilla ei ole mitään yhteyttä sinuun tai sinun havaintoosi.


      • Anonyymi
        Anonyymi kirjoitti:

        "Kyseessä ei ole mikään mielivaltaisesti määritelty käsite. ... Tämä muodostaa lukuisten mahdollisten historioiden verkoston ja aikaa mitataan tuon järjestelmän reaktioiden suhteella."

        Olet määritellyt olevasi sisäinen näkökulma.

        "Kuuluisan kaksoisrakokokeen tulos selitetään monimaailmatulkinnan kautta siten, että historiat voivat reagoida keskenään - ja ne erkanevat peruuttamattomasti vasta, jos niiden yhdistyminen 100% identtiseksi tulevaisuuden tilaksi on mahdotonta."

        Jos haluaa kaksoisrakokokeen tuottavan pisteitä kuvalevylle, niin silloin on välttämätöntä sanoa, että maailmaksi nimitetyt historiat eivät vaikuta toisiinsa enää mitentkään, vaan että piste jää sinne minne se ensin syntyi.

        Tullakseen pisteeksi, pisteen ei tarvitse olla vielä olemassa. Kun sanotaan piste voidaan ajatella, että se on muiden asioiden lisäksi yhden saapuvan hiukkasen aaltofunktion yksi kohta ja sijainti avaruudessa. Kun MWI:ssä sanotaan maailma ja että on korrelaatiota havaitsijan tilan ja saapuvan hiukkasen tilojen välillä, niin tämä piste avaruudessa on se Hilbertin avaruuden suunta, johon havaitsija korreloituu (hänestä näyttää Kööpenhaminalaisesti kuin piste olisi täytetty todennäköisyydellä 1), kun hän sitä myöhemmin pitää tuloksena. Nyt voidaan sanoa, että ei pidä paikkansa että tämä avaruuden piste ja siinä oleva aaltofunktio eri hetkinä ja alkaen alussa todennäköisyydellä 0 olisi reagoinut minkään kanssa. Paitsi aalto tässä pisteessä on reagoinut jotenkin mainitsemattomalla tavalla sellaisen materian kanssa, mitä levyssä on. Väitit kuitenkin, että aaltofunktio reagoisi ajan ja avaruuden yli aaltoon, jonka varsinainen aaltomassa on vasta tulossa kohti levyä, tai että levyllä olevilla pisteillä, jotka seuraavaksi ovat selvästi eri maailmoja, olisi tekemistä keskenään kun niissä on osa aallosta luonaan.

        Ennen saapumista levylle ja mittaukseen ei myöskään tulisi puhua mainittavalla tavalla mistään reagoinnista. Kun kaksoisrakokoe esitetään kahtena palloaaltona, siinä puhutaan tiloista, jotka ovat ikään kuin jaettuna kahtia. Jotkut pitävät näiden tilojen historioita erillisinä ja puhuvat niiden sekoittamisesta. Levyllä meidän tulee kuitenkin jakaa ongelma tiloihin, jotka ovat yksittäisiä pisteitä. Näilläkin pisteillä on historia joka ajan hetkellä. Kun kahdesta pallosta tai mistä hyvänsä tulee funktion amplitudia näiden pisteiden luokse, kyseessä ei ole minkään reagoiminen mihinkään, vaan kyseessä on sellainen odottelu, missä jonkin funktion odotetaan kehittyvän ajassa sellaiseksi, että näiden pisteiden luona funktio on jotain muuta kuin nolla, mikä ei koske niitä kaikkia. Minkä tahansa QM-tulkinnan esitys siitä, mitä mittauksessa tapahtuu, on vaillinainen, eikä sille ole esim. selitystä, joka etenisi tällaisena funktiona ajassa, vaikka MWI sitä haluaisi olevansa. Olisi myös huomattavasti kätevämpää puhua asioista ilman kahta rakoa aaltojen tiellä.

        Äsken olleet palloaallot voivat merkitä jonkun mielestä maailmoja myös, mutta silloin maailmat ovat vain miniskulaarista osatietoa hiukkasen menneistä ajoista (vaikka ovatkin tiloja ja tilat ovat aina yhtä täysiä tiloja kuin muutkin). Lisäksi kukaan havaitsija ei yleensä mittaa kumpaakan näistä maailmoista omakseen. Ja tällaisessa maailmassa maailma ei jakaudu niihin vaan on jäänyt jakautumatta. Normaalissa kokeessa ei pitäisi pitää paikkaansa, että palloaallot olisivat maailmat, jotka olisivat syntyneet hetkeksi ja tuotu yhteen. Missä rako olisi se, missä jako olisi tapahtunut, mutta joka olisi myös tajunnut ettei jaetakaan.

        "Ulkopuolinen näkökulma on sellainen, millä ei ole minkäänlaista kausaalista yhteyttä tuon useita historioita sisältävän järjestelmän kanssa."

        Kyseessä oli linkeissä kietoutunut yhteys ja sen puuttuminen. Kietoutunut yhteys ei ole kausaalinen.

        "Kyllä on määritelty.
        Ulkopuolinen näkökulma on sellainen, joka ei ole missään kausaalisessa yhteydessä (edes hiukkastason reaktioissa) itse systeemin kanssa."

        Nyt kun olet ulkopuolella, etkä ilmeisesti näe aikaa, niin määrittele maailmoita.

        "Käytännön koejärjestely ja tulokset on kuvattu:"

        Artikkeli ei määritellyt useita maailmoita. Mutta käytti suhteellisia tiloja ollessaan systeemin sisällä.

        Tässä on kirjoitettu pidempi review teorialle.
        https://arxiv.org/pdf/1610.04773.pdf

        "Kyseessä oli linkeissä kietoutunut yhteys ja sen puuttuminen. Kietoutunut yhteys ei ole kausaalinen.”

        Totta. Käytin väärää termiä, mutta kun useiden eri hiukkasten tilat kietoutuvat (lomittuvat) keskenään siten, että niitä kuvaa yhteinen aaltofunktio eli mahdollisten tilojen rypäs, niin kyllähän niiden voidaan katsoa oleva tällöin myös kausaalisessa yhteydessä.

        Niin pitkää kuin havainnoitsija ei ole kietoutunut tuohon systeemiin mukaan, niin eri todennäköisyysfunktiot (mahdolliset historiat) voivat reagoida keskenään, mutta kun havaitsija on yhteydessä systeemiin, niin todennäköisyysfunktio aktualisoituu havainnoijan näkökulmasta.


      • Anonyymi
        Anonyymi kirjoitti:

        "Kyseessä oli linkeissä kietoutunut yhteys ja sen puuttuminen. Kietoutunut yhteys ei ole kausaalinen.”

        Totta. Käytin väärää termiä, mutta kun useiden eri hiukkasten tilat kietoutuvat (lomittuvat) keskenään siten, että niitä kuvaa yhteinen aaltofunktio eli mahdollisten tilojen rypäs, niin kyllähän niiden voidaan katsoa oleva tällöin myös kausaalisessa yhteydessä.

        Niin pitkää kuin havainnoitsija ei ole kietoutunut tuohon systeemiin mukaan, niin eri todennäköisyysfunktiot (mahdolliset historiat) voivat reagoida keskenään, mutta kun havaitsija on yhteydessä systeemiin, niin todennäköisyysfunktio aktualisoituu havainnoijan näkökulmasta.

        "Kvanttimekaniikassa MWI-tulkinnassa ei ole mitään tulosta, vaan kaikki mahdolliset historiat ovat olemassa todennäköisyysfunktiona. Tulos aktualisoituu havainnoitsijalle vasta, kun olet kausaalisessa yhteydessä systeemiin, jonka jälkeen sinä olet väistämättä osa yhtä historiaa. Tämä ei tarkoita, etteivätkö muutkin mahdolliset historiat, missä sinä ole, jatkaisi olemassa oloaan ihan yhtä lailla."

        Reagointi, mistä minä kommentoin, ei ole minun puolestani sidottu mihinkään vaihtoehtona olevaan aiheeseen. Eli aktualisoitumisen ja kaikkena-mahdollisena-olemisen välillä ei ole sellaista eroa, etteivätkö molemmat olisi tilanteita ja hetkiä kyseisessä lauseessa ja 'tulos' on sama kuin seuraava hetki, jonkun sanoin kuivaillun hetken jälkeen. Kun olet esittänyt reagointeja, puhut niistä yleensä vain yhdessä tapauksessa, ja lauseeni voidaan osoittaa niille.

        "Se historia, joka on aktualisoitunut sinulle, on vielä määrittelemätön niille, joilla ei ole mitään yhteyttä sinuun tai sinun havaintoosi."

        Siinä käytetään MWI:n sitä tulkintaa, että jokainen joka alkaa asiasta keskustelemaan, on yksi objektin monista tiloista. Tässä ei silti tarvitse tapahtua mitään ennen tähän yhden aktualismin olemassaolon ja muiden aktualismien puuttumiseen päätymistä. Eikä tarvitse esittää mitään lisättävää QM:ään eikä esittää mitään laajaa systeemiä. Jos kirjoittaa qubiteille tilan (000 + 011 + 100 + 111) /2 , ensimmäinen qubitti voi olla sinun kaksi versiotasi. Se ei ole kietoutunut toisiin qubitteihin, mutta nämä ovat kietoutuneet tosiinsa ja ovat kuvitellusti maailmoissa toisiinsa nähden. Aiheeseen liittyen tämä on ajattomasti totetutettavissa oleva tapaus. Lisäksi tästä näkee sen, että jos jonkin arvo on määrittelemätöntä, se lausahdus on täysin määriteltyä aaltofunktiossa ja myös se, että kenestä ja mistä on kyse. Periaatteessa kun qubitti yksi esiintyy superpositio-kubittien kanssa, niin hänellä on täysin valmis ennustus siitä, että mitä ns. määriteltyjä ja klassisia asioita tulee olemaan tai mitä ne jo ovat. Eli hän ei ole mistään informaatiosta varsinaisesti erillään, tai riippuu täysin siitä, mitä kutsuu informaatioksi.

        "Jos mittaat kumman raon läpi elektroni (tms) kulki, niin interferenssi kuviota ei synny. Olet ollut kokeen kanssa kausaalisessa yhteydessä, jolloin havainnoit vain yhden historian - eivätkä mahdolliset potentiaaliset historiat reagoi keskenään. Jos taas et mittaa, minkä raon läpi elektronit menevät, olet systeemin ulkopuolella ja havaitset interferenssi-kuvion - eli kaikkien mahdollisten elektronin rata-historioiden yhteisvaikutuksen."

        Jos on mahdollista tehdä omasta kvanttitilastaan ja saapuvan elektronin kvanttitilasta millainen tahansa, niin MWI voisi väittää, että on olemassa seuraavanlainen todellisuus. Ensin on sellainen tila, missä elektroni jaetaan kahden tilan superpositioon, joka on elektronin koko oikea tila jopa kahden ket-vektorin summana: Psi on L + R eli vasen ja oikea. Tämän jälkeen MWI voisi väittää, että joku ja hänen mahdolliset vektorinsa on korreloitunut vain yhteen tilaan näistä. Silloin selitys nähdylle asialle olisi, että se tulee koska siinä esim. L:n evoluutiota seurataan levyyn asti, koska korreloitumisen takia mitään R:ää ei ole havaitsijalle olemassa. Jotta näin on, niin L on myös normalisoitu todennäköisyydeksi 100 %. Tässä ei väitetä niin, että järjestelmässä ei pitäisi antaa kaiken 'reagoida' eli, että kaikkea todennäköisyysamplitudia, mitä on olemassa Psissä tai kenen tahansa Psissä ei toteutettaisi kaikkialla avaruudessa ja laskettaisi yhteen joka pisteessä. Pikemminkin väite on, että kun korreloitunut havaitsija laskee kaiken yhteen, hän lisää L:ään pelkkiä nollia, koska teorian mukaan se miksi häntä siinä sanotaan on kertonut kaiken muun nollalla.

        Jos kirjoittaisit tällä tavalla, niin sinulla olisi vielä se ongelma, että et voi muuttaa tätä sellaiseen näkökulmaan, missä on sekä L ja R läsnä eli johonkin isompaan näkökulmaan, ja sen lisäksi väittää, että siinäkin näkökulmassa sama olisi kuvailtu sanoilla 'se ja se ei reagoi'. Jotain sinun kuitenkin pitää siinä sanoa siitä, mikä erottaa mittaavat tutkijat niistä jotka eivät mittaa. MWI voi yrittää muodostaa suuremman tilan ja säilyä täysin puhtaana tilana kokonaisuutta varten, mutta jos tarkastelu on vähän pienempi, niin mainitsemasi kausaalisen tapahtuman jälkeen, joka muuttaa systeemin eri muotoon kuin ilman L:n (maailma h1) R:n (maailma h2) tuntemista (h on havaitsija), on erilainen vain sekoitetun tilan tiheysmatriisin osalta, missä lukee 'L h1:htä varten' ja 'R h2:hta varten' samalla, kun sekä L ja R ovat sitä, mitä ne olivat aiemmin.

        1


      • Anonyymi
        Anonyymi kirjoitti:

        "Kvanttimekaniikassa MWI-tulkinnassa ei ole mitään tulosta, vaan kaikki mahdolliset historiat ovat olemassa todennäköisyysfunktiona. Tulos aktualisoituu havainnoitsijalle vasta, kun olet kausaalisessa yhteydessä systeemiin, jonka jälkeen sinä olet väistämättä osa yhtä historiaa. Tämä ei tarkoita, etteivätkö muutkin mahdolliset historiat, missä sinä ole, jatkaisi olemassa oloaan ihan yhtä lailla."

        Reagointi, mistä minä kommentoin, ei ole minun puolestani sidottu mihinkään vaihtoehtona olevaan aiheeseen. Eli aktualisoitumisen ja kaikkena-mahdollisena-olemisen välillä ei ole sellaista eroa, etteivätkö molemmat olisi tilanteita ja hetkiä kyseisessä lauseessa ja 'tulos' on sama kuin seuraava hetki, jonkun sanoin kuivaillun hetken jälkeen. Kun olet esittänyt reagointeja, puhut niistä yleensä vain yhdessä tapauksessa, ja lauseeni voidaan osoittaa niille.

        "Se historia, joka on aktualisoitunut sinulle, on vielä määrittelemätön niille, joilla ei ole mitään yhteyttä sinuun tai sinun havaintoosi."

        Siinä käytetään MWI:n sitä tulkintaa, että jokainen joka alkaa asiasta keskustelemaan, on yksi objektin monista tiloista. Tässä ei silti tarvitse tapahtua mitään ennen tähän yhden aktualismin olemassaolon ja muiden aktualismien puuttumiseen päätymistä. Eikä tarvitse esittää mitään lisättävää QM:ään eikä esittää mitään laajaa systeemiä. Jos kirjoittaa qubiteille tilan (000 011 100 111) /2 , ensimmäinen qubitti voi olla sinun kaksi versiotasi. Se ei ole kietoutunut toisiin qubitteihin, mutta nämä ovat kietoutuneet tosiinsa ja ovat kuvitellusti maailmoissa toisiinsa nähden. Aiheeseen liittyen tämä on ajattomasti totetutettavissa oleva tapaus. Lisäksi tästä näkee sen, että jos jonkin arvo on määrittelemätöntä, se lausahdus on täysin määriteltyä aaltofunktiossa ja myös se, että kenestä ja mistä on kyse. Periaatteessa kun qubitti yksi esiintyy superpositio-kubittien kanssa, niin hänellä on täysin valmis ennustus siitä, että mitä ns. määriteltyjä ja klassisia asioita tulee olemaan tai mitä ne jo ovat. Eli hän ei ole mistään informaatiosta varsinaisesti erillään, tai riippuu täysin siitä, mitä kutsuu informaatioksi.

        "Jos mittaat kumman raon läpi elektroni (tms) kulki, niin interferenssi kuviota ei synny. Olet ollut kokeen kanssa kausaalisessa yhteydessä, jolloin havainnoit vain yhden historian - eivätkä mahdolliset potentiaaliset historiat reagoi keskenään. Jos taas et mittaa, minkä raon läpi elektronit menevät, olet systeemin ulkopuolella ja havaitset interferenssi-kuvion - eli kaikkien mahdollisten elektronin rata-historioiden yhteisvaikutuksen."

        Jos on mahdollista tehdä omasta kvanttitilastaan ja saapuvan elektronin kvanttitilasta millainen tahansa, niin MWI voisi väittää, että on olemassa seuraavanlainen todellisuus. Ensin on sellainen tila, missä elektroni jaetaan kahden tilan superpositioon, joka on elektronin koko oikea tila jopa kahden ket-vektorin summana: Psi on L R eli vasen ja oikea. Tämän jälkeen MWI voisi väittää, että joku ja hänen mahdolliset vektorinsa on korreloitunut vain yhteen tilaan näistä. Silloin selitys nähdylle asialle olisi, että se tulee koska siinä esim. L:n evoluutiota seurataan levyyn asti, koska korreloitumisen takia mitään R:ää ei ole havaitsijalle olemassa. Jotta näin on, niin L on myös normalisoitu todennäköisyydeksi 100 %. Tässä ei väitetä niin, että järjestelmässä ei pitäisi antaa kaiken 'reagoida' eli, että kaikkea todennäköisyysamplitudia, mitä on olemassa Psissä tai kenen tahansa Psissä ei toteutettaisi kaikkialla avaruudessa ja laskettaisi yhteen joka pisteessä. Pikemminkin väite on, että kun korreloitunut havaitsija laskee kaiken yhteen, hän lisää L:ään pelkkiä nollia, koska teorian mukaan se miksi häntä siinä sanotaan on kertonut kaiken muun nollalla.

        Jos kirjoittaisit tällä tavalla, niin sinulla olisi vielä se ongelma, että et voi muuttaa tätä sellaiseen näkökulmaan, missä on sekä L ja R läsnä eli johonkin isompaan näkökulmaan, ja sen lisäksi väittää, että siinäkin näkökulmassa sama olisi kuvailtu sanoilla 'se ja se ei reagoi'. Jotain sinun kuitenkin pitää siinä sanoa siitä, mikä erottaa mittaavat tutkijat niistä jotka eivät mittaa. MWI voi yrittää muodostaa suuremman tilan ja säilyä täysin puhtaana tilana kokonaisuutta varten, mutta jos tarkastelu on vähän pienempi, niin mainitsemasi kausaalisen tapahtuman jälkeen, joka muuttaa systeemin eri muotoon kuin ilman L:n (maailma h1) R:n (maailma h2) tuntemista (h on havaitsija), on erilainen vain sekoitetun tilan tiheysmatriisin osalta, missä lukee 'L h1:htä varten' ja 'R h2:hta varten' samalla, kun sekä L ja R ovat sitä, mitä ne olivat aiemmin.

        1

        Viimeinen lause on juuri sellainen missä oletetaan, että tiloihin saisi korreloitua ilman, että niissä tapahtuu muutoksia. Kuitenkin kaikki toteutuneet tavat mitata, mitä kautta elektroni meni, ovat jotain missä raon taakse päästetty elektroni ei ole koskaan entisensä aaltona (tai samanlainen kuin yhden raon kokeessa yhden raon jälkeen) vaikka se edelleen jonain suurena aaltona pysyisi. Elektronin käynti tietyssä raossa voidaan aiheuttaa sille siten, että se menettää energiaa raon aineeseen, mitä kutsutaan myös epäelastiseksi sironnaksi.

        https://www.researchgate.net/publication/273580024_Elastic_and_inelastic_electrons_in_the_double-slit_experiment_A_variant_of_Feynman's_which-way_set-up

        Kun lasketaan mitä tapahtuu, jos olisi kaksi epäelastista rakoa tai vain toinen epäelastisena, niin tämä tavallaan tapahtuu tietyllä tavalla MWI-hengessä ja ulkopuolisen näkökulmalla, koska siinä otetaan molemmat todennäköisyysaallot ja lasketaan ne avaruudessa yhteen kuin reagoiden. Tällöin nähdään, että interferenssikuvio (jota pyytäisin ajattelemaan sellaisena kuviona, minkä näkisi, jos kaikki maailmat olisivat läpinäkyviä, mutta näyttäisivät, mikä yksittäisen elektronin levypiste oli niissä; tai sitten vaan usean elektronin jälkeen nähty kuva, jota yllä tituleerasin kaikkien maailmoiden näkemiseksi myös) voi sallia interferenssiä niin paljon kuin tavallisestikin, mutta sitä ei tule samoihin paikkoihin ja kauas sivuille asti, koska epäelastisen sironnan jälkeen todennäköisyysaaltojen vaiheet ovat siirtyneet.

        MWI:tä voi kuitenkin harjoittaa vielä lisää siten, että sanoo muodostuvan maailman 1 sitä varten, että epäelastinen sironta oli vasemmassa raossa. Tai puhua ylipäänsä raon aineesta, eli sanoa että vasemman raon aine on yhdeltä usean tilan tilaltaan korreloitunut siihen, että energia ja muut sironnan aiheuttamat muutokset aineessa menivät sinne, mikä tarkoittaa samaa eli on korrelaatiossa sen kanssa, että oikea rako korreloituu siihen, että sinne ei tullut energiaa, minkä lisäksi koko todellisuudessa on vielä sellainen maailma, että nämä ovat toisin päin. Mutta missään tapauksessa, missä rako 1 on jotain, niin toinen rako ei ole epävarma superpositio, eikä ole elektronikaan. (Koko raon aine on kuin h, mutta jako h1 tai h2 ei merkitse mitään. MWI:n periaate on pikemminkin, että kun olisi klassisuuden muodostamisesta kyse, on mahdotonta tietää tai edes kuvailla sen merkitystä, että mikä h olet, koska ne ovat lian samat keskenään ja edellyttäisivät miljardien ja miljardien hiukkasten tilojen tuntemista. Ongelman yksinkertaistus kahden objektin neljään tilaan tarkoittaa samaa kuin jättäisi kaiken mahdollisen huomioimatta ja sisällyttäisi sen yhteen h1:hteen, mikä tällöin käsittää kummankin raon aineet ja kuvailee niiden molempien sain-tai-en-saanut-energiaa -vaihtoehdot yhdellä kertaa.)

        Vaiheiden tuottamat kuvat ja alussa olleet mielikuvitusmittaus kuvat vertautuvat toisiinsa siten, että jos osaisit ottaa miljoonan vaihekoe-elektronin joukosta pois ne jotka kadottivat energiaa L:ssä, niin jäljelle jäisi samanlainen piirtolevy, kuin jos L olisi tukittu kokonaan miljoonan elektronin ajaksi. Tämä on olettaen, että epäelastisuus on niin tehokasta (tapahtuu aina), että sen jälkeen raon takana ei ole koskaan mahdollista superpositiota sekä vasemmasta että oikeasta. Kun L on tukittu, jäljelle jää sellainen epäelastinen tai tavallinen R-aalto, jota vektori-R:n kanssa korreloituneet maailmat näkevät. Tosin R:n kanssa pitäisi myös korreloitua miljoona kertaa peräkkäin saadakseen sen näkyviin sulkematta L.ää koskaan.

        Artikkelin pohjalta ajateltu koe on pikemminkin jonkinlainen yhteistodennäköisyyden funktion piirtyminen mielessä raon jälkeiseen avaruuteen. Se on pikemminkin matemaattinen johdannainen, eikä siinä ole enää jäljellä erillisiä historioita (*). Tällaisena voisi toimia mielummin aiemmin mainittu matriisi. Sanotaan siitä vielä, että sekoitetun tilan matriisin muodossa h1 ajautuu erilleen R:stä siten, että rivit, joilla on h1R, ovat diagonaalilta nollia. Tämän aikaansaanti tai kumoaminen ei ole kuvailtavissa samoin kuin R:n ja L:n summa yhdessä objektissa. Mikään ei voi tulla lasketuksi yhteen sen matriisin kanssa, mikä nyt esiintyy vain h1L ja h2R -matrioisina, ja kasvattaa riviä h1R, ellei ole jo ajateltu, että h1R-tulosta on jo olemassa eikä se missään poissa ollutkaan. Sen sijaan on tapahduttava lisää vuorovaikutusta, joka on h ja Psi -tilojen välillä ja joka vaikkapa kääntää matriisia.

        (*) Tämän kappaleen jälkeen ei myöskään Psi:ssä ole mitään useita historioita, vaan erillistäminen tai eri asioina pidettyjen historioiden ei-erillisyyden perusta on monta objektia, missä ne sitten varmasti lukevat. Ei-erillisyys on se, että sekä h1R ja h1L ovat nollasta poikkeavia, minkä lisäksi ne ovat toivottavasti kvanttitiloja.

        2


      • Anonyymi
        Anonyymi kirjoitti:

        Viimeinen lause on juuri sellainen missä oletetaan, että tiloihin saisi korreloitua ilman, että niissä tapahtuu muutoksia. Kuitenkin kaikki toteutuneet tavat mitata, mitä kautta elektroni meni, ovat jotain missä raon taakse päästetty elektroni ei ole koskaan entisensä aaltona (tai samanlainen kuin yhden raon kokeessa yhden raon jälkeen) vaikka se edelleen jonain suurena aaltona pysyisi. Elektronin käynti tietyssä raossa voidaan aiheuttaa sille siten, että se menettää energiaa raon aineeseen, mitä kutsutaan myös epäelastiseksi sironnaksi.

        https://www.researchgate.net/publication/273580024_Elastic_and_inelastic_electrons_in_the_double-slit_experiment_A_variant_of_Feynman's_which-way_set-up

        Kun lasketaan mitä tapahtuu, jos olisi kaksi epäelastista rakoa tai vain toinen epäelastisena, niin tämä tavallaan tapahtuu tietyllä tavalla MWI-hengessä ja ulkopuolisen näkökulmalla, koska siinä otetaan molemmat todennäköisyysaallot ja lasketaan ne avaruudessa yhteen kuin reagoiden. Tällöin nähdään, että interferenssikuvio (jota pyytäisin ajattelemaan sellaisena kuviona, minkä näkisi, jos kaikki maailmat olisivat läpinäkyviä, mutta näyttäisivät, mikä yksittäisen elektronin levypiste oli niissä; tai sitten vaan usean elektronin jälkeen nähty kuva, jota yllä tituleerasin kaikkien maailmoiden näkemiseksi myös) voi sallia interferenssiä niin paljon kuin tavallisestikin, mutta sitä ei tule samoihin paikkoihin ja kauas sivuille asti, koska epäelastisen sironnan jälkeen todennäköisyysaaltojen vaiheet ovat siirtyneet.

        MWI:tä voi kuitenkin harjoittaa vielä lisää siten, että sanoo muodostuvan maailman 1 sitä varten, että epäelastinen sironta oli vasemmassa raossa. Tai puhua ylipäänsä raon aineesta, eli sanoa että vasemman raon aine on yhdeltä usean tilan tilaltaan korreloitunut siihen, että energia ja muut sironnan aiheuttamat muutokset aineessa menivät sinne, mikä tarkoittaa samaa eli on korrelaatiossa sen kanssa, että oikea rako korreloituu siihen, että sinne ei tullut energiaa, minkä lisäksi koko todellisuudessa on vielä sellainen maailma, että nämä ovat toisin päin. Mutta missään tapauksessa, missä rako 1 on jotain, niin toinen rako ei ole epävarma superpositio, eikä ole elektronikaan. (Koko raon aine on kuin h, mutta jako h1 tai h2 ei merkitse mitään. MWI:n periaate on pikemminkin, että kun olisi klassisuuden muodostamisesta kyse, on mahdotonta tietää tai edes kuvailla sen merkitystä, että mikä h olet, koska ne ovat lian samat keskenään ja edellyttäisivät miljardien ja miljardien hiukkasten tilojen tuntemista. Ongelman yksinkertaistus kahden objektin neljään tilaan tarkoittaa samaa kuin jättäisi kaiken mahdollisen huomioimatta ja sisällyttäisi sen yhteen h1:hteen, mikä tällöin käsittää kummankin raon aineet ja kuvailee niiden molempien sain-tai-en-saanut-energiaa -vaihtoehdot yhdellä kertaa.)

        Vaiheiden tuottamat kuvat ja alussa olleet mielikuvitusmittaus kuvat vertautuvat toisiinsa siten, että jos osaisit ottaa miljoonan vaihekoe-elektronin joukosta pois ne jotka kadottivat energiaa L:ssä, niin jäljelle jäisi samanlainen piirtolevy, kuin jos L olisi tukittu kokonaan miljoonan elektronin ajaksi. Tämä on olettaen, että epäelastisuus on niin tehokasta (tapahtuu aina), että sen jälkeen raon takana ei ole koskaan mahdollista superpositiota sekä vasemmasta että oikeasta. Kun L on tukittu, jäljelle jää sellainen epäelastinen tai tavallinen R-aalto, jota vektori-R:n kanssa korreloituneet maailmat näkevät. Tosin R:n kanssa pitäisi myös korreloitua miljoona kertaa peräkkäin saadakseen sen näkyviin sulkematta L.ää koskaan.

        Artikkelin pohjalta ajateltu koe on pikemminkin jonkinlainen yhteistodennäköisyyden funktion piirtyminen mielessä raon jälkeiseen avaruuteen. Se on pikemminkin matemaattinen johdannainen, eikä siinä ole enää jäljellä erillisiä historioita (*). Tällaisena voisi toimia mielummin aiemmin mainittu matriisi. Sanotaan siitä vielä, että sekoitetun tilan matriisin muodossa h1 ajautuu erilleen R:stä siten, että rivit, joilla on h1R, ovat diagonaalilta nollia. Tämän aikaansaanti tai kumoaminen ei ole kuvailtavissa samoin kuin R:n ja L:n summa yhdessä objektissa. Mikään ei voi tulla lasketuksi yhteen sen matriisin kanssa, mikä nyt esiintyy vain h1L ja h2R -matrioisina, ja kasvattaa riviä h1R, ellei ole jo ajateltu, että h1R-tulosta on jo olemassa eikä se missään poissa ollutkaan. Sen sijaan on tapahduttava lisää vuorovaikutusta, joka on h ja Psi -tilojen välillä ja joka vaikkapa kääntää matriisia.

        (*) Tämän kappaleen jälkeen ei myöskään Psi:ssä ole mitään useita historioita, vaan erillistäminen tai eri asioina pidettyjen historioiden ei-erillisyyden perusta on monta objektia, missä ne sitten varmasti lukevat. Ei-erillisyys on se, että sekä h1R ja h1L ovat nollasta poikkeavia, minkä lisäksi ne ovat toivottavasti kvanttitiloja.

        2

        "(Tämä on vastaava ilmiö kuin edellä mainitun g-kertoimen johdossa Feynmanin diagrammesta. Näemme kaikkien mahdollisten historioiden yhteisvaikutuksen.)"

        Yksi vastaväite on vielä se, että yksi tärkeä vastaavuus puuttuu, ja se on se, että Feynmanin diagrammin lopputilaa (jonka todennäköisyyttä itseään ei väitetä 1:ksi vaan joksikin pieneksi mitä siinä ollaan laskemassa näillä) voi pitää niin satunnaisena kuin voi vain olla, eli se on kuin analogia jostakin mikä olisi tullut tietyksi lopputilaksi vain Schrödingerin yhtälön kaltaisen evoluution myötä (pienessä suljetussa ja eristetyssä systeemissä, missä ei voi sanoa, että MWI:n universaalinen aaltofunktio hoitaisi sille dekoherenssia jostain ympäristöstä) eikä ole koskaan kuvitellutkaan vielä törmäävänsä johonkin levyyn, niinkuin kaksoisrakokokeessa.

        "MWI:ssä todennäköisyysfunktio kuvaa mahdollisten historioiden nippua. Todennäköisfunktion aallot ovat eri potentiaalisten historioiden todennäköisyystiheyksiä,..."

        Tämän tulisi ehkä tarkoittaa koko todellisuuden aaltofunktiota. En ole tähän esitykseen tyytyväinen, koska en usko siihen täysin, että aaltofunktio osattaisiin lukea siten kuin se olisi useiden historioiden nippu. Varsinkaan vaikeissa tapauksissa, missä on asioita, mitä sanotaan maailmaksi, ja missä aaltofunktion pitäisi silloin olla vastaus mittausongelmaan. Sen sijaan se niputtuu kyllä joksikin.

        "... ja koska nämä aallot eli hiukkasen mahdolliset radat (historiat) kykenevät reagoimaan keskenään, voimme tulkita, että nämä kaikki historiat ovat todellisia."

        Tulkitsemislupaa ei voi saada noin. Everett sanoi sen kuin siten, että koska yhden objektin yhden tilan mielestä toisen objektin tila voi näyttää siltä, kuin toinen tila olisi klassinen, niin sen voi tulkita siten, että kummankaan objektin tila ei olisi koskaan lakannut olemasta kaikissa tiloissaan. Argumentoiko hän tilannetta, joka menee aivan eri suuntaan kuin sinulla ja saa saman tuloksen?

        ...

        'Todellinen' sana viittaa tässä ilmeisesti koko todellisuuteen eikä yhteen maailmaan. Yllä olet kuitenkin kirjoittanut, että faktuaalinen reagointi (vs. 'reagointi ehkä') on konditionaalista. Jos joku on yhteydessä systeemiin, niin otat reagoinnin pois. Tällöin tässä lukee, että MWI:n perustulkinnan saa tehdä vain tapauksissa, missä ei ole mittaajaa. Tai vähintään, että se mitä tapahtuu ilman mittaajia, perustelee miksi MWI:n voi tehdä.

        Et ole myöskään maininnut määritelmissä kovin usein sitä, että MWI:n pitää muodostaa yhdistettyjä aaltofunktoita ja tehdä mittaajana toimivasta muusta aineesta sen osa, toisin kuin romahdustulkinnat. Tällä yhdistetyllä aaltofunktoilla on myös asiaa muuttua historian aikana ja olla monta koherenttia tai epäkoherenttia historiaa. Ja koska siinä on vähintään kaksi asiaa, niin kahden asian välisestä vuorovaikutuksesta eli niiden reagoinnista toisiinsa voidaan puhua ihan eri sävyyn, mutta edelleen jossakin mikä on aaltofunktio (ket-vektorin asemassa eikä matemaattisen 3D-kompleksifunktion).

        "Monimaailmatulkinnasta seuraa myös ennustuksia. Jos pystyisit eristämään itsesi taustalevystä täysin, niin ettei elektronin siihen edellä aktualisoitunut sijainti vaikuttaisi systeemiisi (sinuun) mitenkään (edes epäsuorasti), niin periaatteessa aktualisoitumista ei tapahtuisi ja kaikkien mahdollisten historioiden ketju eläisi sinun näkökulmasta vieläkin pidemmälle."

        Tämä ei ole varsinaisesti MWI-ennustus. Tekeväthän Feynmanin diagrammitkin noin, että jos lähetät kvanttitilan päin kvanttitilaa, niin kvanttitilana ne pysyvät. Mitä pienemmän kvanttisysteemin esität laskutehtävänä, niin sitä selvempää se on, että se on ja jatkuu näin. Mutta samalla sitä vähemmän sillä on mitään MWI:ssä kiinnostavaa ja sitä näkyväksi/ajateltavaksi tekevää efektiä, kuten dekoherenssi.

        "Totta. Käytin väärää termiä, mutta kun useiden eri hiukkasten tilat kietoutuvat (lomittuvat) keskenään siten, että niitä kuvaa yhteinen aaltofunktio eli mahdollisten tilojen rypäs, niin kyllähän niiden voidaan katsoa oleva tällöin myös kausaalisessa yhteydessä."

        Yhteys asioiden välillä pitää katsoa yhteydestä eikä siitä, mitä asiat ovat. Yhteys ei ole määritelty siten, että yhteyden voisi tuntea vain kun hiukkaset ovat toteuttaneet yksittäisen vaihtoehdon. Onhan MWI:ssäkin pystyttävä ymmärtämään aika joukko mahdollisia vaihtoehtoja, jotta mikään historia (pienemmän systeemin, missä voi sanoa olevan kaiken yhtäkkiä aivan päinvastoin kietoutumisten kanssa eikä niinkuin yhdellä hetkellä päätettiin) ei jäisi siitä puuttumaan. Sen jälkeen jokaisesta yhteydestä pystyy toteamaan, onko yhteys kausaalinen vai epäkausaalinen.

        3


      • Anonyymi
        Anonyymi kirjoitti:

        "(Tämä on vastaava ilmiö kuin edellä mainitun g-kertoimen johdossa Feynmanin diagrammesta. Näemme kaikkien mahdollisten historioiden yhteisvaikutuksen.)"

        Yksi vastaväite on vielä se, että yksi tärkeä vastaavuus puuttuu, ja se on se, että Feynmanin diagrammin lopputilaa (jonka todennäköisyyttä itseään ei väitetä 1:ksi vaan joksikin pieneksi mitä siinä ollaan laskemassa näillä) voi pitää niin satunnaisena kuin voi vain olla, eli se on kuin analogia jostakin mikä olisi tullut tietyksi lopputilaksi vain Schrödingerin yhtälön kaltaisen evoluution myötä (pienessä suljetussa ja eristetyssä systeemissä, missä ei voi sanoa, että MWI:n universaalinen aaltofunktio hoitaisi sille dekoherenssia jostain ympäristöstä) eikä ole koskaan kuvitellutkaan vielä törmäävänsä johonkin levyyn, niinkuin kaksoisrakokokeessa.

        "MWI:ssä todennäköisyysfunktio kuvaa mahdollisten historioiden nippua. Todennäköisfunktion aallot ovat eri potentiaalisten historioiden todennäköisyystiheyksiä,..."

        Tämän tulisi ehkä tarkoittaa koko todellisuuden aaltofunktiota. En ole tähän esitykseen tyytyväinen, koska en usko siihen täysin, että aaltofunktio osattaisiin lukea siten kuin se olisi useiden historioiden nippu. Varsinkaan vaikeissa tapauksissa, missä on asioita, mitä sanotaan maailmaksi, ja missä aaltofunktion pitäisi silloin olla vastaus mittausongelmaan. Sen sijaan se niputtuu kyllä joksikin.

        "... ja koska nämä aallot eli hiukkasen mahdolliset radat (historiat) kykenevät reagoimaan keskenään, voimme tulkita, että nämä kaikki historiat ovat todellisia."

        Tulkitsemislupaa ei voi saada noin. Everett sanoi sen kuin siten, että koska yhden objektin yhden tilan mielestä toisen objektin tila voi näyttää siltä, kuin toinen tila olisi klassinen, niin sen voi tulkita siten, että kummankaan objektin tila ei olisi koskaan lakannut olemasta kaikissa tiloissaan. Argumentoiko hän tilannetta, joka menee aivan eri suuntaan kuin sinulla ja saa saman tuloksen?

        ...

        'Todellinen' sana viittaa tässä ilmeisesti koko todellisuuteen eikä yhteen maailmaan. Yllä olet kuitenkin kirjoittanut, että faktuaalinen reagointi (vs. 'reagointi ehkä') on konditionaalista. Jos joku on yhteydessä systeemiin, niin otat reagoinnin pois. Tällöin tässä lukee, että MWI:n perustulkinnan saa tehdä vain tapauksissa, missä ei ole mittaajaa. Tai vähintään, että se mitä tapahtuu ilman mittaajia, perustelee miksi MWI:n voi tehdä.

        Et ole myöskään maininnut määritelmissä kovin usein sitä, että MWI:n pitää muodostaa yhdistettyjä aaltofunktoita ja tehdä mittaajana toimivasta muusta aineesta sen osa, toisin kuin romahdustulkinnat. Tällä yhdistetyllä aaltofunktoilla on myös asiaa muuttua historian aikana ja olla monta koherenttia tai epäkoherenttia historiaa. Ja koska siinä on vähintään kaksi asiaa, niin kahden asian välisestä vuorovaikutuksesta eli niiden reagoinnista toisiinsa voidaan puhua ihan eri sävyyn, mutta edelleen jossakin mikä on aaltofunktio (ket-vektorin asemassa eikä matemaattisen 3D-kompleksifunktion).

        "Monimaailmatulkinnasta seuraa myös ennustuksia. Jos pystyisit eristämään itsesi taustalevystä täysin, niin ettei elektronin siihen edellä aktualisoitunut sijainti vaikuttaisi systeemiisi (sinuun) mitenkään (edes epäsuorasti), niin periaatteessa aktualisoitumista ei tapahtuisi ja kaikkien mahdollisten historioiden ketju eläisi sinun näkökulmasta vieläkin pidemmälle."

        Tämä ei ole varsinaisesti MWI-ennustus. Tekeväthän Feynmanin diagrammitkin noin, että jos lähetät kvanttitilan päin kvanttitilaa, niin kvanttitilana ne pysyvät. Mitä pienemmän kvanttisysteemin esität laskutehtävänä, niin sitä selvempää se on, että se on ja jatkuu näin. Mutta samalla sitä vähemmän sillä on mitään MWI:ssä kiinnostavaa ja sitä näkyväksi/ajateltavaksi tekevää efektiä, kuten dekoherenssi.

        "Totta. Käytin väärää termiä, mutta kun useiden eri hiukkasten tilat kietoutuvat (lomittuvat) keskenään siten, että niitä kuvaa yhteinen aaltofunktio eli mahdollisten tilojen rypäs, niin kyllähän niiden voidaan katsoa oleva tällöin myös kausaalisessa yhteydessä."

        Yhteys asioiden välillä pitää katsoa yhteydestä eikä siitä, mitä asiat ovat. Yhteys ei ole määritelty siten, että yhteyden voisi tuntea vain kun hiukkaset ovat toteuttaneet yksittäisen vaihtoehdon. Onhan MWI:ssäkin pystyttävä ymmärtämään aika joukko mahdollisia vaihtoehtoja, jotta mikään historia (pienemmän systeemin, missä voi sanoa olevan kaiken yhtäkkiä aivan päinvastoin kietoutumisten kanssa eikä niinkuin yhdellä hetkellä päätettiin) ei jäisi siitä puuttumaan. Sen jälkeen jokaisesta yhteydestä pystyy toteamaan, onko yhteys kausaalinen vai epäkausaalinen.

        3

        "Niin pitkää kuin havainnoitsija ei ole kietoutunut tuohon systeemiin mukaan, niin eri todennäköisyysfunktiot (mahdolliset historiat) voivat reagoida keskenään, mutta kun havaitsija on yhteydessä systeemiin, niin todennäköisyysfunktio aktualisoituu havainnoijan näkökulmasta."

        Historian tarkka määritelmä, joka sopii keskusteltavaksi ja pysyttelee tarkastelemassa kaikkea, on esim. sellainen summa kubitteja, missä yksi kubitti, joka esittää näkökulmaansa on kaikissa qubiteissa sama. Eli esim. 011 + 000. Kun reagointi tarkoittaa interferenssejä, niin esim. näille kolmelle kubitille, mikään ei keskeytä kubittien summaamista yhteen, olivat ne mitä hyvänsä. Kun reagointi tarkoittaa sitä, että eri kubitit näistä eli esim. toinen ja kolmas vuorovaikuttavat keskenään, niin mikään ei estä sitä tapahtumasta. Eli esim. ensimmäisen kubitin esittely systeemille ja tämän kolmikon pitäminen uutena systeeminä ei vaikuta siihen pitäisikö kahden alkuperäisen olla edelleen vuorovaiktusuksessa tai ei. Eikä mikään tila, minkä nämä kolme kubittia ottavat, voi lopettaa kahdella niistä olevaa vuorovaikutusta.

        Kun kietoudutaan vuorovaikutusta sisältävään systeemiin (mitä kaikissa systeemeissä ei olisi ollut) havaitsijan kuvitellaan olevan sellainen, mikä tekee
        systeemistä
        000+011+100+111 /2
        systeemin
        000+111 / sqrt 2 tai yhden toisista

        Tässä ei tarvitse todennäköisesti tapahtua yhtään vuorovaikutusta toisen ja kolmannen välillä, vaan havaitsijan mittaus on riittävä muodostamaan keskustellun yhteyden. Voisit koettaa käyttää versiotasi MWI:stä selittämään juuri jotain tällaista, että miten kietoutuminen on löydetty hiukkasen 2 ja 3 välillä siinä maailmassa, missä nyt ollaan. Ja ilmaise siinä juuri se, että miten havaitsija mittaamalla hiukkasen 2 saa laitettua itsensä ja sellaisen hiukkasen 3, jota hän ei ole mitannut, omaan maailmaansa. Vieläpä silloin kun 2. ja 3. eivät ole kausaalisessa yhteydessä, vaan ne on erotettu toisistaan ja .3 on viety liian kauas.

        4


      • Anonyymi
        Anonyymi kirjoitti:

        "Niin pitkää kuin havainnoitsija ei ole kietoutunut tuohon systeemiin mukaan, niin eri todennäköisyysfunktiot (mahdolliset historiat) voivat reagoida keskenään, mutta kun havaitsija on yhteydessä systeemiin, niin todennäköisyysfunktio aktualisoituu havainnoijan näkökulmasta."

        Historian tarkka määritelmä, joka sopii keskusteltavaksi ja pysyttelee tarkastelemassa kaikkea, on esim. sellainen summa kubitteja, missä yksi kubitti, joka esittää näkökulmaansa on kaikissa qubiteissa sama. Eli esim. 011 000. Kun reagointi tarkoittaa interferenssejä, niin esim. näille kolmelle kubitille, mikään ei keskeytä kubittien summaamista yhteen, olivat ne mitä hyvänsä. Kun reagointi tarkoittaa sitä, että eri kubitit näistä eli esim. toinen ja kolmas vuorovaikuttavat keskenään, niin mikään ei estä sitä tapahtumasta. Eli esim. ensimmäisen kubitin esittely systeemille ja tämän kolmikon pitäminen uutena systeeminä ei vaikuta siihen pitäisikö kahden alkuperäisen olla edelleen vuorovaiktusuksessa tai ei. Eikä mikään tila, minkä nämä kolme kubittia ottavat, voi lopettaa kahdella niistä olevaa vuorovaikutusta.

        Kun kietoudutaan vuorovaikutusta sisältävään systeemiin (mitä kaikissa systeemeissä ei olisi ollut) havaitsijan kuvitellaan olevan sellainen, mikä tekee
        systeemistä
        000 011 100 111 /2
        systeemin
        000 111 / sqrt 2 tai yhden toisista

        Tässä ei tarvitse todennäköisesti tapahtua yhtään vuorovaikutusta toisen ja kolmannen välillä, vaan havaitsijan mittaus on riittävä muodostamaan keskustellun yhteyden. Voisit koettaa käyttää versiotasi MWI:stä selittämään juuri jotain tällaista, että miten kietoutuminen on löydetty hiukkasen 2 ja 3 välillä siinä maailmassa, missä nyt ollaan. Ja ilmaise siinä juuri se, että miten havaitsija mittaamalla hiukkasen 2 saa laitettua itsensä ja sellaisen hiukkasen 3, jota hän ei ole mitannut, omaan maailmaansa. Vieläpä silloin kun 2. ja 3. eivät ole kausaalisessa yhteydessä, vaan ne on erotettu toisistaan ja .3 on viety liian kauas.

        4

        Historian tarkasta määritelmästä ei tullut vielä valmista tuolla, mitä edellä oli. Täytyy myös olla laskematta liian montaa maailmaa siinä tapauksessa, että henkilön tila h1 + h2 on tensoritulo
        (h1 + h2) o (mikä hyvänsä tila superpositioineen tai ei superpositioineen).

        Määritelmästä maailmojen määrien laskemiseksi tulee sellaisia keskusteluun sopivia lauseita kuten, 'jos maailmoja on määritelmän mukaan enemmän kuin yksi, silloin niiden määrilmän kaltaisesti kirjoitetuissa lauseissa ei ole laskettuna yhteen tiloja, joilla on eri ominaistiloja, kun taas maailmoja ollessa yksi, niin näin on'.

        5


    • Anonyymi

      Aika on nopeuden nimittäjä.

    • Anonyymi

      Aika on fysiikan perussuure, jonka yksikkö on sekunti.

      • Anonyymi

        Kvanttimittausteorian mukaan aika ei ylipäänsä ole fysikaalinen suure.


    • Anonyymi

      Aika on alkuräjähdyksen oheistuote.

    • Anonyymi

      Palloheilurissa pallon energia vain muuttaa muotoaan liike-energista potentiaalienergiaksi. Prosessi etenee nopeammin tai hitaammin riippuen langan pituudesta ja pallon massasta.

      Jos ei tapahdu mitään, niin ei ole aikaakaan.

      • Anonyymi

        Aika ja energia ovat sodoksissa toisiinsa. Fotonin energia on
        E = h f = h/T
        T = h/E
        h on Planckin vakio. T on värähdysaika eli fotoniin liityvä ajan "kvantti". Mitä suurempi fotonin energia, sitä lyhyempi ajan kvantti siihen liittyy.


      • Anonyymi

        Kun esitetään kaava muodossa ∆E ∆t = h, ollaan jo epämääräisyysperiaatteen tuntumassa.


      • Anonyymi

        Epämääräisyysperiaate on koko kvanttifysiikan kulmakivi.

        Jos ajan käsitteestä luovuttaisiin, niin sillä olisi järisyttävät vaikutukset nykyfysiikkaan perustuvaan maailmankuvaan.


      • Anonyymi
        Anonyymi kirjoitti:

        Kun esitetään kaava muodossa ∆E ∆t = h, ollaan jo epämääräisyysperiaatteen tuntumassa.

        Vaikka kaava löytyy jopa 1980-luvun lukion fysiikan oppikirjoissa, moderni kvanttiteoria ei sitä hyväksy. 1920-luvulla tätä kaavaa tarvittiin ensimmäisessä teoriassa, jolla selitettiin antimaterian olemassaoloa.

        Heisenbergin epätarkkuusperiaate on määritetty kvanttisuureiden x ja p välillä. Niillä on minimikommutaattori. Näitä suureita ei voi tuosta vain muokata E:ksi ja t:ksi, kuten klassisen mekaniikan vastaavia suureita.


    Ketjusta on poistettu 3 sääntöjenvastaista viestiä.

    Luetuimmat keskustelut

    1. Naiset miltä kiihottuminen teissä tuntuu

      Kun miehellä tulee seisokki ja ja sellainen kihmelöinti sinne niin mitä naisessa köy? :)
      Sinkut
      110
      8316
    2. Olet sä kyllä

      ihme nainen. Mikä on tuo sun viehätysvoiman salaisuus?
      Ikävä
      39
      2529
    3. Teuvo Hakkaraisesta tulee eurovaalien ääniharava

      Persuissa harmitellaan omaa tyhmyyttä
      Maailman menoa
      121
      2253
    4. Hiljaiset hyvästit?

      Vai mikä on :( oonko sanonut jotain vai mitä?
      Ikävä
      22
      1929
    5. Miksi kohtelit minua kuin tyhmää koiraa?

      Rakastin sinua mutta kohtelit huonosti. Tuntuu ala-arvoiselta. Miksi kuvittelin että joku kohtelisi minua reilusti. Hais
      Särkynyt sydän
      15
      1654
    6. Turha mun on yrittää saada yhteyttä

      Oot mikä oot ja se siitä
      Suhteet
      11
      1489
    7. Kyllä poisto toimii

      Esitin illan suussa kysymyksen, joka koska palstalla riehuvaa häirikköä ja tiedustelin, eikö sitä saa julistettua pannaa
      80 plus
      16
      1452
    8. "Joka miekkaan tarttuu, se siihen hukkuu"..

      "Joka miekkaan tarttuu, se siihen hukkuu".. Näin puhui jo aikoinaan Jeesus, kun yksi hänen opetuslapsistaan löi miekalla
      Yhteiskunta
      14
      1389
    9. Näkymätöntä porukkaa vai ei

      Mon asuu yksin. Mitas mieltä ootte ?
      Ikävä
      15
      1272
    10. Kristityt "pyhät"

      Painukaa helvettiin, mä tulen sinne kans. Luetaan sitten raamattua niin Saatanallisesti. Ehkä Piru osaa opetta?!.
      Kristinusko
      12
      1183
    Aihe